Thread Rating:
  • 0 Vote(s) - 0 Average
  • 1
  • 2
  • 3
  • 4
  • 5
lots of questions from classified section of this - anothergoljafan
#1
SN Questions
1- 57 y/o male is on hemodialysis for CRF. Which of the ffg metabolic derangements can be anticipated?
A- hypercalcemia
B- hypophosphatemia
C- osteomalacia
D- Vit. D excess
E- Hypoparathyroidism

Answer is c- osteomalacia.
Kidneys failure to excrete phosphate àhyperphosphatemia & fail to synthesize 1,25(OH)2D3. then Vit. D def. àimpaired intestinal ca absorption.
Phosphate retention, defective intestinal absorption & skeletal resistance to PTH à hypocalcemia à secondary hyperparathyroidism à worsens hyperphosphatemia by increasing the Ph. release from boneà renal osteodystrophy



2- 50 y/o woman with a Hx of essential HTN, presents to ER with severe headache, nausea, vomiting and photophobia. On P/E, BP= 160/100 mmHg, she™s confused with nuchal rigidity without focal neurologic signs.
** What™s the Dx?
A- hemorrhagic stroke
B- ruptured cerebral aneurysm
C- meningitis
D- ischemic cerebrovascular accident
E- TIA


** Dx is best confirmed with?
A- LP
B- Doppler US of carotid artries
C- Head MRI
D- Head CT followed by cerebral angiography
E- EEG


** What™s the next most important step in Mx?
A- urgent surgery with aneurysm clipping
B- admission to ICU, close monitoring and Tx of HTN
C- admission to ICU, close monitoring and IV antibiotics
D- anticoagulation & antiplatlete therapy
E- serial LP to drain CSF

Answers are B, D, A
most likley the pt. ruptured cerebral anurysm, as pt. is confused, no focal signs present and pt. has nuchal rigidity, could be and most likely due to the blood in the CSF--> meningeal irritation, etc..
although you also see nuchal rigidity w/ meningitis, here you have confusion as well
although they give us a nice high B.P of 160/100, and we might want to choose stroke over cerebral anurysm, given the rest of the prez. here anurysm is more likly, as well as you would expect to see neuro focal findings w/ stroke, not present here
ischemic cerebrovascular accident --> expect to see some focal signs as well as may not have nuchal rigidity
TIA, usually transient and other symptoms present , not seen here


3- During an elective laparascopic cholecystectomy, the patient developes a sudden onset drop in BP, atrial desaturation and an increase in ventilatory pressure. What™s the most appropriate step of Mx?
A- an IV fluid bolous
B- decompression of the pneumoperitoneum
C- inserting a chest tube
D- re-evaluating the position of the endotracheal tube & portable CXR
E- aborting the procedure & converting to an open cholecystectomy

Answer is C, coz patient has developed tension pneumothorax.




4- 2 wk old boy is brought for check up. He™s been doing well at home. Results of his newborn screen shows he has sickle cell dis. What™s the most important step in his Mx?
A- avoiding heat exposure
B- immunization with pneumococcal vaccine
C- folic acid supplements
D- iron supplements
E- prophylactic penicillin


** Six months later, the same patient returns with a 3 day Hx of lethargy & fever with rhinorrhea and cough. On P/E, he is pale, tacchycardic with a LUQ mass. Hgb= 4, Plt= 100000, WBC= 15000 with 50% segmented neut. , reticulocyte count=15%. What™s the Dx?
A- Acute chest syn.
B- Acute splenic sequestration
C- Aplastic crisis
D- Intussusception
E- Vaso-occlusive crisis


Answer is E & B. because he™s at high risk of infection, especially with encapsulated organisms and penicillin will dramatically decrease this risk. Pneumococcal vaccine is not effective in neonates. It™s usually given at 2 yrs age.



5- A 52 y/o man receives a preoperative evaluation before an alective surgery. He is asymptomatic with a normal P/E, but he is noted to have a Hgb= 10.8, Hct= 33, MCV= 70, RBC= 6.1x1000/ microlit. What™s the most likely Dx?
A- Sickle cell dis.
B- Iron def. Anemia
C- Alpha thalassemia major
D- Beta thalassemia minor
E- Anemia of chronic dis.

Answer is D. low MCV is either IDA or beta-thal. BUT, high no. of RBCs is characteristic of beta-thal. In IDA, RBC count is below normal.
pt. has hypocromic, microcytic anemia,..as well as increased retic. count--> increased RBC's, ..this last feature makes the better choice here to be thalassemia, as oposed to iron def. for example
iron def. seems to be avery good choice, as one should think anemia in older MAN , is due to cancer-COLON ca , until proven otherwise..however you would expect to see low reticulocytes count w/ iron def. anemia.
note- elevated retic. counts--> seen in thalassemia/hemoglobinopathy,..like SS anemia

6- match the clinical description with the most likely organism:

a- Strep. Pneumoniae
b- Staph. Aureus
c- Strep. Viridans
d- Providentia stuartii
e- Actinomyces israelii
f- Hemophilus ducreyi
g- Neisseria meningitides
h- Listeria monocytogene

1- 30 y/o female with MVP, MR develops fever, anorexia & weight loss after a dental procedure

2- 80 y/o male hospitalized for hip Fx, has foley cath. in place, develops shaking chills, fever & hypotension.

3- young man develops painless, fluctuant, purplish lesion over mandible, after several weeks cutaneous fistula is noted.

4- sickle cell patient presents with high fever, toxicity signs of pneumonia & stiff neck.


Answers:
1- c, 2- d, 3- e, 4-a





7- 65 y/o male with Hx of DM & cardiomyopathy, presents with severe knee pain. On P/E, knee is swollen, red & tender. Knee X-Ray shows linear clcification.
*** Dx is best made by:
A- Serum uric acid
B- Serum calcium
C- Arthrocentesis & identification of birefringent rhomboid crystals
D- Rheumatoid factor

*** Further workup inthis patient should include evaluation for:
A- Renal dis.
B- Hemochromatosis
C- PUD
D- Lyme dis.

Answers are C & B. Acute monoarticular arthritis in association with linear calcification in the cartilage of knee maqkes the Dx of pseudogout which is positive for birefringent crystals in joint fluid. Pseudogout maybe associated with hemochromatosis.



8- 65 y/o woman with a 12 Hx of symmetric polyarthritis, presents with splenomegaly, ulcers on lat. Malleoli, synovitis of wrists, shoulders and knees, and no hepatomegaly.
Lab results : WBC=2500, RF= 1:4096, this patients WBC diff is most likely to show what ?
A- pancytopenia
B- lymphopenia
C- granulocytopenia
D- lymphocytosis
E- basophilia

Answer is C. case of felty™s syn. ( RA+ splenomegaly+ leukopenia ), the mech. Of granulocytopenia is poorly understood.


9- A patient with low grade fever & weight loss has poor excursion on the Rt. side of chest with decreased fremitus, flatness to percussion and decreased breath sounds. Trachea is deviated to the Lt. What™s the most likely Dx?
A- Pneumothorax
B- Pleural effusion secondary to histoplasmosis
C- Consolidated pneumonia
D- Atelectasis

Answer is B. Physical findings all consistent with pleural effusion, which in large amount can shift trachea to the Lt. in pneumothorax hyperresonance of the affected side is present. Atelectasis on the Rt would shift trachea to the Rt.




10- A patient have an unexpected high value for diffusing capacity, this finding is most consistent with which of the ffg?
A- Anemia
B- Cystic fibrosis
C- Emphysema
D- Intrapulmonary hemorrhage

Answer is D. Decreased diffusing capacity is seen in: primary parenchymal disorder, anemia & removal of lung tissue.
Increased value is seen in : polycythemia, CHF & intrapulmonary hemorrhage.



11- A 65 y/o man is admitted to the hospital with anginal chest pain. His general health has been excellent, although he has had a multinodular goiter for many years. He had a series of thyroid function tests 4 weeks before admission, and the results are :
Serum T4 8.0 µg/dL
Free T4 index 8.0
Serum T3 152 ng/dL
Serum thyrotropin (TSH) 0.7 µU/mL

A MI is ruled out, but chest pain continues. A coronary arteriogram shows a 90% stenosis of the left main coronary artery. A coronary artery bypass graft is done. The patient has an uneventful postoperative course and is discharged on the seventh postoperative day. One month later, he is readmitted in atrial fibrillation with a rapid ventricular response. Repeat thyroid function testing shows the following:
Serum T4 15.0 µg/dL
Free T4 index 15.8
Serum T3 220 ng/dL
Serum TSH <0.01 mU/µL

What is the most likely Dx?
A. Graves' disease
B. Stress-induced hyperthyroidism
C. Iodine-induced hyperthyroidism
D. Silent thyroiditis
Answer is C. The natural history of multinodular goiters is slow growth and gradual decrease in thyrotropin (TSH), reflecting increasing thyroid hormone production. This progression occurs over years to decades, however. Many patients with multinodular goiters have autonomous areas within their thyroid. This patient had normal thyroid function 1 month before admission. However, his serum TSH level was near the lower limits of normal, suggesting the possibility of autonomous thyroid function.
When patients with multinodular goiters are exposed to excess iodine, severe hyperthyroidism may occur. This is known as iodine-induced hyperthyroidism or the Jod-Basedow phenomenon. When iodine supplementation is introduced into areas of iodine deficiency, iodine-induced hyperthyroidism may occur in patients with multinodular goiters. Iodine-induced hyperthyroidism may occur in nonendemic goiter areas as well, often with devastating consequences.
The high iodine content of the dye used for the cardiac catheterization undoubtedly precipitated the hyperthyroidism in this patient. The onset of hyperthyroidism may be delayed for several weeks to months after the iodide exposure.
Although other causes of hyperthyroidism are possible, none is as likely as this scenario. When patients with multinodular goiter must be exposed to excess iodine (for example, during cardiac catheterization, computed tomographic [CT] scan with contrast medium, or amiodarone therapy), premedication with antithyroid drugs (methimazole or propylthiouracil) should be considered.


12- A 43 y/o woman complains of itching that keeps her awake at night. Physical examination is normal, except for the liver, which is felt 7 cm below the right costal margin.The blood count is normal; the results of serum chemistry tests are as follows:
¢ Creatinine 0.8 mg/dL
¢ Bilirubin 0.6 mg/dL
¢ Alanine aminotransferase 78 U/L
¢ Albumin 4.2 g/dL
¢ Alkaline phosphatase 450 U/L
Which test would you order next in order to diagnose the underlying disorder?
(A) Serum protein electrophoresis
(B) Anti¬smooth-muscle antibody
© Antimitochondrial antibody
(D) Technetium-99m liver-spleen scan
(E) ERCP

Answer is C. This is the classic description of primary biliary cirrhosis. Itching is the most common specific symptom of early primary biliary cirrhosis. Approximately 70% of affected patients have enlarged livers. The best screening test for suspected primary biliary cirrhosis is the antimitochondrial antibody test. It is positive in 95% of affected patients and has a 98% specificity if newer enzyme-linked immunosorbent assay (ELISA) tests are used. Serum protein electrophoresis might show a diffuse increase in immunoglobulins. However, this finding is nonspecific and may be found in many chronic liver diseases. Anti¬smooth-muscle antibody tests are positive in some patients with autoimmune chronic hepatitis. However, the test is nonspecific and not terribly useful. The sulfur colloid technetium liver-spleen scan is useful for detecting portal hypertension and hypersplenism. However, it is not specific and would not be helpful in diagnosing primary biliary cirrhosis. Endoscopic retrograde cholangiopancreatography (ERCP) is typically normal in patients with primary biliary cirrhosis. Its only role in the diagnosis of primary biliary cirrhosis is in the patient who presents with a similar syndrome but who has a negative antimitochondrial antibody test. ERCP would then be performed to look for other causes of disease such as primary sclerosing cholangitis.



13- A patient presents to your office with complaints of severe earache and decreased hearing on the left side. Examination confirms left otitis media. You prescribe an oral cephalosporin and an analgesic. The next day the patient is brought to the emergency room with a rash and moderate bronchospasm. Reviewing his office record you are horrified to discover that he is known to have a severe allergic reaction to penicillin.
What should you tell the patient at this point?
(A) Apologize and explain what happened.
(B) Apologize and offer to compensate him financially.
© Attribute the rash and the bronchospasm to his infection.
(D) Explain to the patient that he is allergic to cephalosporin as well as penicillin.
(E) No explanations are necessary.
Answer is A. Medical errors are common but are infrequently reported to patients. The fear is that acknowledging a mistake may lead to litigation. Nevertheless, there is no excuse for a physician not to admit a mistake, which is an inevitable part of medical practice. Apologizing and explaining is not only ethical, but also has been shown to decrease the risk of litigation. There is no need or obligation to offer financial compensation. Trying to attribute your mistake to other causes, or attempting to blame the patient for your negligence is not only unethical, but will increase your risk of litigation. Finally, nothing will anger a patient more than no explanation at all.



14- A 29 y/o woman presents with an exacerbation of her asthma. She is 11 weeks pregnant. She has mild intermittent asthma and usually takes a ß-agonist as needed. She has one 4-year-old child who is in day care and has had a recent upper respiratory tract infection. She has a dry cough, clear nasal discharge, myalgias, and fatigue.
On physical examination, she is talking in full sentences and has normal tympanic membranes, mildly erythematous oropharynx without exudates, no adenopathy; she has positive wheezing bilaterally. The peak flow is 300 mL; her usual result is 390 mL. Pulse oximetry is 93% on room air.Which of the following is indicated in the management of this patient?
(A) Amoxicillin
(B) Theophylline
© Prednisone
(D) Montelukast
(E) Flunisolide
Answer is E. Treatment principles for asthma in the nonpregnant patient apply also to the pregnant patient. She has mild intermittent asthma with an exacerbation, for which therapy with an inhaled glucocorticoid is an appropriate choice. Use of inhaled glucocorticoids is safe in pregnancy.
Asthma in pregnancy has a variable course, with one third of patients getting better, one third getting worse, and one third staying the same. This patient has had mild symptoms and now has an exacerbation related either to an upper respiratory tract infection, or to worsening of her asthma because of her pregnancy. She has no signs of streptococcal pharyngitis, sinusitis, bacterial tracheobronchitis, or pneumonia; antibiotics should therefore not be given. Prednisone therapy should be reserved for further symptoms. Theophylline can be used in pregnancy with close monitoring, but it is not the next drug of choice. Montelukast may be used in pregnancy, but should be reserved for progressive symptoms.
The patient should continue therapy with ß-agonists. Use of a peak flow meter at home is integral to her management; she should be instructed and encouraged in its use.




15- A 30-year-old woman presents because of 6 months of amenorrhea. She had regular menses starting at age 13 years and is otherwise healthy. Menses gradually became irregular over the past 5 years, in spite of relatively stable weight and activity. She has no acne or hirsutism. The results of recent laboratory tests included normal serum thyroid-stimulating hormone and prolactin levels and a negative serum pregnancy test.
What is the next most useful diagnostic test for the evaluation of secondary amenorrhea in this patient?
(A) Measurement of LH
(B) Measurement of FSH
© Pelvic US
(D) Measurement of serum estradiol
(E) MRI of the pituitary gland
Answer is B. In this patient, the negative pregnancy test, and normal serum TSH and prolactin levels exclude the most common causes of secondary amenorrhea. The remaining causes include hypothalamic amenorrhea and ovarian failure. Of the tests listed, serum FSH is the most likely to be diagnostic of a reproductive endocrine abnormality, as it has a longer half life than luteinizing hormone and becomes elevated relatively early in the process of ovarian aging. Luteinizing hormone is less useful because of its greater variability across the normal menstrual cycle (including the dramatic, often 10-fold, increase at the mid-cycle surge), its more striking 1 to 2 hourly pulsatility, and its elevation in polycystic ovary syndrome as well as ovarian failure. Serum estradiol is frequently in the normal range in various reproductive disorders and therefore rarely useful. MRI of the pituitary gland is not indicated unless other tests are normal and a hypothalamic cause of amenorrhea needs to be ruled out. Finally, pelvic ultrasound can show the presence of ovaries, their size, and the uterine morphology and endometrial thickness, but cannot diagnose the cause of secondary amenorrhea. It may be more useful in the evaluation of primary amenorrhea, if the physical examination is inadequate to confirm the presence of intact ovaries and a uterus, or in the evaluation of excessive menstrual bleeding, when it can identify endometrial thickening, polyps, and/or uterine leiomyomas. A medroxyprogesterone challenge test can show that the uterine anatomy and vaginal outflow track are normal, which is suggested by the history of previously normal menstrual cycles. In addition, the challenge test provides some information about recent estrogen exposure. However, it cannot be diagnostic in estrogen deficiency.


16- A 2 y/o has a chronic cough. CXR reveals hyperinflation of the left hemithorax. Bilateral decubitus views showed that the right lung becomes appropriately atelectatic however, the left side shows no change in appearance with decubitus positioning. What™s the most likely diagnosis?

Asymmetric hyperinflation suggests the possibility of an aspirated foreign body in this two year old infant. These findings suggest the diagnosis of an aspirated foreign body or a possible obstruction of the left main stem bronchus from a central etiology. Bronchoscopy is both diagnostic and therapeutic.



17- match this drugs with their associated syndrome:
A- barbiturates
B- Ecstasy
C- Inhalants
D- Marijuana
E- Methamphetamine
F- PCP

1- Severe encephalopathy
2- Lung cancer
3- Rhabdomyolysis during intoxication
4- Wanting to touch/be touched during intoxication
5- Seizures during withdrawal
6- œ Swiss cheese œ appearance on functional brain imaging
A-5, B-4, C-1, D- 2, E-6, F-3


18- 72 y/o man with Hx of UTI & CHF is admitted for sepsis & pulmonary edema. He™s treated with clindamycin, tobramycin & IV furosemide. After 4 days, sepsis signs are improves but BUN= 60 & Cr.= 5 mg/dl. BP=125/75 , PR=72( no postural changes ).
1-What™s the most likely cause of his renal dysfunction?
A- Prerenal azotemia
B- ATN
C- Interstitial nephritis
D- Hypercalcemic nephropathy

2- What™s the best way to confirm the Dx?
A- Urine Na of 25 mEq/L
B- Renal tubular epithelial cells & muddy brown casts in sediment
C- Negative US
D- Abnormalities of medulla in IVP

Aswers are B & B. There™s no clinical evidence of prerenal azotemia, so the most likely Dx is ATN due to toxicity with aminoglycoside. Urine sediment in ATN is abnormal and shows renal tubular epithelial cells, debris & muddy brown casts. Since patient has been getting diuretics, high urine Na is less specific.

19- 70 y/o man with Hx of COPD, presents with worsening SOB of the last couple days. He™s coughing yellow-colored sputum and gets no result from his beta 2 agonist & ipratropium aerosolized pumps. On P/E, RR=40, HR=110, BP= 155/85, he™s afebrile and using his accessory muscles for respiration. Also inspiratory & expiratory diffuse wheezing on both sides of his lungs are heard. What™s the most likely Dx?
A- Acute exacerbation of COPD
B- Alfa-1 antitrypsin def.
C- Chronic bronchitis
D- Exacerbation of asthma
E- Pneumonia
Answer is A- Acute exacerbation of COPD, it occurs when patient develops acute onset of marked dyspnea & tachypnea with use of accessory mucles with no response to medication.


20- 73 y/o male with Hx of HTN, presents with short episode of Lt. sided weakness & slurred speech. Also he has a Hx of 3 brief episodes of sudden Rt. eye vision impairment in the last month.
1- What™s the best next diagnostic test?
A- Cerebral MRI
B- Holter monitoring
C- Visual evoked responses
D- Carotid artry doppler US
E- Conventional cerebral angiography
2- Episodes of visual loss are related to:
A- Retinal vein thrombosis
B- Central retinal A. ischemia
C- Post. cerebral A. ischemia
D- Middle cerebral A. ischemia
E- Post. ciliary A. ischemia
Answers are D & B. This is a classicd case of extracranial internal carotid A. dis. which include episodes of ipsilat. transient monocular blindness ( amaurosis fugax ) & contralat. TIA consisting of motor weekness. The most appropriate test to confirm the Dx of carotid stenosis is Doppler us. The mech. of transient monocular blindness is embolism to the central retinal A. or one of its branches.

21- A 5 y/o presents to the ER 3 h after a possible button battery ingestion. The patient is in no acute distress, vital signs are stable, and examination is benign. CXRshows what appears to be a small button battery in the stomach. Which of the following is the MOST appropriate next action?
(A) Upper GI series to further delineate the exact location of the foreign body
(B) Attempt battery removal by the Foley balloon catheter technique
© Immediate GI consultation for endoscopic removal
(D) Immediate surgical consultation
(E) Discharge to home with parental observation and weekly radiographs

The answer is E. Button batteries lodged in the esophagus require emergencyremoval to avoid esophageal burns and perforation. If the button battery has passed the esophagus and the patient is asymptomatic, home observation with serial x-rays toensure passage through the pylorus is the appropriate course of action. Most button batteriesthat have passed the esophagus will transit through the entire body within 24 to 48 h without difficulty. If the battery is of large diameter and the patient is younger than 6
years, the battery is less likely to pass, and endoscopic retrieval is the preferred treatment.


22- A 45 yo male with a long Hx of alcohol use and presumptive gastritis presents to the with sudden onset of severe abdominal pain and vomiting. V/S: BP=110/60, HR=110/min , T=101°F, RR= 30/minhe™s diaphoretic with epigastric tenderness and mild guarding. Lab data: WBC= 30,000, amylase =2,000. CXR shows a small amount of free air under the diaphragm. What is the MOST likely diagnosis?
(A) Acute pancreatitis with associated Mallory- Weiss syndrome
(B) Acute pancreatitis with associated Boerhaave™s syndrome
© Acute pancreatitis with associated enzymatic destruction of bowel wall
(D) Acute pancreatitis secondary to anterior duodenal ulcer perforation
(E) Acute pancreatitis secondary to posterior duodenal ulcer perforation

answer is E. Because the pancreas adheres to the posterior duodenum, ruptured posterior duodenal ulcers generally penetrate into the pancreas rather than perforate into the free peritoneum. Anterior ulcers are more likely to perforate into the peritoneal cavity. A Mallory-Weiss tear of the esophageal wall usually presents with
symptoms similar to reflux esophagitis and causes moderate, self-limited bleeding. Patients with Boerhaave™s syndrome rapidly deteriorate to a state of shock and septicemia
due to a malignant mediastinitis.


23- A 55 y/o male without significant medical history presents with LLQ pain and constipation. V/S: T= 100.5°F, PR= 85, BP=150/80, RR=12. P/E is unremarkable except for mild LLQ tenderness without guarding and rebound. Rectal examination shows heme-negative stool and no tenderness.Lab results are within normal limits except for WBC=13,000 with a left shift. Which of the following would be the MOST appropriate management for this patient?
(A) Prompt surgical evaluation in the ED
(B) Emergent upper GI series
© Emergent barium enema
(D) Discharge to home with bowel rest and oral antibiotics
(E) Discharge home with repeat abdominal examination in 12 h or sooner if worse

The answer is D. The patient described in the scenario most likely has diverticulitis. Patients with localized pain and no signs and symptoms of peritonitis or systemic infection do not require hospitalization. Outpatient management consists of bowel rest, broad-spectrum oral antibiotic therapy, and close follow-up.


24- Which of the following etiologic agents is the MOST common cause of infection in liver transplant patients?
(A) Candida
(B) Cytomegalovirus
© Herpes simplex virus
(D) P. carinii
(E) L. monocytogenes

The answer is B. Complications of infections in liver transplant patients account for nearly 90 percent of deaths. The most common infectious agent after transplantation is Cytomegalovirus (CMV. Reported occurrence ranges from 23 to 85 percent of all liver transplant patients. Fortunately, CMV is rarely fatal.



25- A 56 y/o heavy alcohol user male presents with vomiting blood for several hours. BP=90/60, PR=110, RR=16, and T=98°F. Placement of an NGT shows active bright red bleeding. Which of the following is the treatment of choice?
(A) Tamponade with a Sengstaken-Blakemore tube
(B) Therapeutic upper GI endoscopy
© Octreotide infusion
(D) Vasopressin infusion
(E) Immediate referral for surgical intervention

The answer is B. GI bleeding is a common problem seen in the ED and is potentially life threatening. For patients with significant active bleeding, emergency endoscopy is the treatment of choice. Esophageal varices can be treated endoscopically with either band ligation or sclerotherapy, resulting in control of acute bleeding in up to 90 percent of patients. Hemostasis can be achieved with nonvariceal sources of bleeding as well. Drug therapy with both octreotide and somatostatin reduces bleeding from both varices and PUD and is a useful adjunct to endoscopy. Vasopressin therapy has largely been discontinued due to a high rate of adverse effects including hypertension, arrhythmias, myocardial ischemia, and decreased cardiac output. Balloon tamponade can be used as a temporizing measure to control bleeding by placing direct pressure on the gastric and esophageal mucosa. However, it is frequently associated with complications, including mucosal ulceration, esophageal or gastric rupture, asphyxiation from dislodged balloons, and aspiration pneumonia. Although it is appropriate to make a surgeon aware of a critical patient, emergency surgical intervention is indicated only in those patients who fail endoscopic hemostasis and medical therapy.



*** 36 y/o woman with meno/metrorrhagia, inlarged uterus, which of the ffg would tell the severity of this condition?
a- CBC
b- Hysteroscopy
c- US
d- Pelvic exam

It™s US, when severity of the condn is asked, it need not necessarily mean
bleeding (submucosal and intramural) it could be pressure sym also(subserosal), hence US. to add, since 36 yr most common is fibroids since adenomyosis in the premenopausal, tho' both can have meno metrorrhagia.


*** what™s the most effective way to prevent kidney stones?
a- exercise
b- diet
c- hydration
d- periodic U/A

*** 25 y/o female G1P0, 16 wks of gestation, presents with vaginal bleeding & cramps. No products of conception is expelled yet. What™s the next step of Mx?
a- Adimt to hospital, observation & monitoring
b- Bed rest at home
c- Admit to hospital, D&C

*** Which of the ffg is a contraindication for vaginal delivery after previous c/s ?
a- Low segment transverse uterine incision
b- breech
c- clinically adequate pelvis
d- placenta previa
a. repeat cs possible, unless longitudinal incision, not done nowadays.
b. vaginal can be done.
c. prerequisit for vaginal.
d. regardless of previous delivery, since the incision would be on the low implanted placenta- torrential bleeding unless the (lower placental edge is more than 2cm from the internal os and it the first
delivery without previous CS
answer is d.

*** 23 y/o female with primary amenorrhea, on P/E breats are abesnt, but uterus is present, what test will u orderfirst ?
a- FSH
b- Karyotype
c- Testosterone
d- estrogen

FSH. Good explanation in first aid..



*** Post MI pulmonary edema, what do you do next?

reduction of pulmonary venous return (preload reduction) and reduction of systemic vascular resistance (afterload reduction) with diuretics ( furesmide ) & morphine sulfate, in a case of hypotension give inotropic agents, supplemental O2.


*** 50 y/o male with known Hx of TB, presents with shoertness of breath, CXR shows massive Rt. side pleural effusion, what™s the next step?
a- CT
b- Thoracocentesis
c- Pericardiectomy
d- etc.

b- thoracocentesis


*** in coarctation of aorta:
1- peripheral resistance a- increase b-decrease
2- pulmonary resistance a- increase b- decrease
1- decrease, 2- increase


*** to prevent hepatitis B in a IV drug abuser, what would you do?
a- HBV vaccine
b- HBV Ig
c- Both
d- etc


*** patient presents with trauma to base of neck, damage to which structure causes most severe compromise of cardiorespiratory system?
a- thoracic aorta
b- jugular vein
c- esophagus
d- SVC


19 y/o with a small lump in her Lt./ breast is very concerned that it™s malignant cancer, work up & Bx shows it™s benign, but she is still extremely worried inspite of reassurance by her physician. Which of the ffg is the best Tx in this case?
A- a careful explanation of benign nature of the complaint
B- Use of benzodiazepine
C- Skillful physician reassurance
D- Use of placebo medication
E- Psychotherapy to explore her current life circumstances

Answer is E. case of hypochondriasis which is usually becomes evident during psychological stress.


Which of the ffg risk factors are related to breast carcinoma?
A- Obesity, late manopause, first pregnancy > 35 yr, smoking, high fat diet
B- Obesity, late menopause, low fiber diet, smoking
C- First pregnancy>35 yr, obesity, smoking, family Hx, age
D- Low fiber diet, age, smoking, first pregnancy> 35 yr, late menopause
E- Age, obesity, late menopause, first pregnancy>35 yr, low fiber, high fat diet

Answer is E. Smoking is not a risk factor. The most important risk factors are:
- Age
- Family Hx
- Nulliparous state
- Early menarche
- Late menopause
- Hx of contralat. breast cancer
- First pregnancy > 35 yr
- High fat diet
- Low fiber diet


2 y/o boy who palys in a sandbox in a nursery that has couple cats for children to play with, presents with wheezing, hepatosplenomegaly & peripheral blood eosinophilia. What™s the Dx?
A- Pinworm infestations
B- Loffler™s syn.
C- Ascariasis
D- Visceral larva migrans
E- Strongyloidiasis

Answer is D. Visceral larva migrans is caused by Toxocara larva. It™s most common in children 1-4 y/o especially who have close contacts with digs & cats. Sandboxea are common places for both pets & children. Sx are: fever, hepatosplenomegaly, wheezing, pulmonaer dis. & eosinophilia



17 y/o woman G1P0 with 10 yr Hx of asthma undergoes pulmonary evaluation & spirometery. She doesn™t have any respiratory complaints or Sx. Which of the ffg regarding her status during pregnancy is true?
A- RR increases
B- Vital capacity decreases
C- Minute ventilation increases
D- Functional residual capacity remains unchanged
E- Tidal volume decreased

Answer is C. since Px does™t have any respiratory problem, she™ll undergo the normal changes in respiratory physiology that occur during pregnancy:
- RR is unchanged
- TV is increased
- Minute ventilation is increased
- VC remains unchanged
- FRC is decreased ( uterus elevates the resting position of diaphragm )


35 y/o woman with major depressive disorder, single episode has responded well to imipramine after 1 month of Tx. Which of the ffg is the most appropriate next step?
A- Continue imipramine for 6 months
B- Continue imipramine indefenitly
C- Gradual imipramine decrease till she™s medication free, unless depression occurs
D- Stop imipramine immediately
E- Switch to fluoxetine

Answer is A. Maintanace therapy after response to antidepressants should be continued for 6 months after initial response.




14 y/o girl presents with trouble concentrating at school just three months after witnessing her best friend being shot to death by another classmate. Ever since that "awful day" she has been very "sad" and withdrawn, often sitting on her bed, staring at the wall "for hours." She has frequent crying spells and refuses to play with friends or participate in her normal after-school activities. Her appetite has "dwindled down to nothing" and she feels very guilty that she survived the shooting. On further questioning, she reluctantly admits that she has constant thoughts of "joining her friend." She did not receive any counseling after the incident.
*** What™s the most likely Dx?
A. adjustment disorder
B. brief psychotic disorder
C. dysthymic disorder
D. major depressive disorder
E. normal grief
F. PTSD

*** What™s the most important q to ask at this time?
A. "Are you hearing voices?"
B. "Do you have any friends that you feel comfortable talking to at school?"
C. "Do you wear a helmet when you ride your bicycle?"
D. "Can you describe your typical weekly alcohol intake?"
E. "Have you thought of the means by which you can 'join your friend'?"


1- The correct answer is D. This patient most likely has major depressive disorder. To meet the criteria for this disorder a patient must exhibit a 2-week history of a distinct change in mood or a loss of interest or pleasure, along with at least 4 of the following: a decreased appetite and weight loss, difficulty sleeping, psychomotor retardation or agitation, fatigue, feelings of worthlessness or guilt, an inability to concentrate, and suicidal ideation. The symptoms must cause functional impairment. ( Kaplan, step 3 samples )

2- The correct answer is E. Since she has already told you that she has suicidal ideation ("joining her friend"), it is very important to ask if she has "thought of the means by which" she can "join her friend" or has made any preparatory actions.




A 64 y/o married man has been diagnosed to be HIV positive. He comes in to the office to discuss the results and begs you not to tell his wife, who is also your patient. He says that hearing what he has been in involved in "will kill her." You remind him that HIV may also "kill her." The most appropriate first step is to:

A. contact his wife anonymously and tell her that she has been exposed to HIV
B. contact the appropriate government health agency and report your findings
C. promise him that as his doctor you will respect his privacy and maintain confidentiality
D. promise him that you will keep the results confidential if he agrees to use condoms with his wife
E. try to persuade him to voluntarily discuss the issue with his wife

The correct answer is E. Physicians must violate confidentiality and warn third persons about the danger of HIV infection if the patient is unwilling to inform the person himself. Before informing third parties, the physician should try to do everything possible to persuade the patient to voluntarily discuss the issue with their partner. If the physician believes that an individual may really be saved from a deadly infection, confidentiality should be violated.



A 4 y/o boy presents with fever, irritability, and erythema of the hands and feet for the past week. His mother has been giving him aspirin to reduce his temperature. P/E on admission showed a T=39.7 C (103.4 F), bilateral conjunctival injection, an enlarged right-sided cervical lymph node (1.8-cm), fissured lips, a red tongue with red papillae, pharyngeal hyperemia, erythematous and edematous palms and soles, and a confluent, blanching erythematous rash on the trunk. IV fluids were started, the aspirin therapy was continued. Laboratory studies show ESR= 28mm/h Plt= 490,000/mm3. The patient is extremely uncomfortable and now shows desquamation of the fingers and toes. The most appropriate therapy at this time is:

A. corticosteroids
B. ibuprofen
C. intravenous gammaglobulin
D. oxacillin
E. penicillin V

The correct answer is C. This patient most likely has Kawasaki disease, which is treated with aspirin and intravenous gammaglobulin. The disease is characterized by a high fever for longer than 5 days, bilateral conjunctival injection, fissured lips, a "strawberry tongue", mucosal change in the oral pharynx, erythematous and edematous palms and soles with desquamation, a polymorphous rash, cervical lymphadenopathy, an elevated erythrocyte sedimentation rate, and thrombocytosis. The most important complication is coronary artery aneurysms, which may be prevented by early treatment with aspirin and intravenous gammaglobulin. An echocardiogram is necessary to evaluate cardiac involvement.




47 y/o woman with a Hx of similar attacks of epigastric abdominal pain in the past was admitted to the hospital with a Dx of gallstone pancreatitis. She was NPO and IV fluid started. On the evening of admission day, the patient is noted to have T=103.4 F. Her BP & HR are within normal range. Her abdomen is diffusely tender to palpation with guarding. What™s the most appropriate management at this time?

A. draw blood cultures and await results
B. draw blood cultures and initiate ampicillin, gentamicin, and metronidazole therapy
C. draw blood, urine, and sputum cultures and await results
D. obtain an urgent abdominal CT scan
E. start ampicillin, gentamicin, and metronidazole therapy

The correct answer is B. The most appropriate management at this time is to draw blood cultures and initiate ampicillin, gentamicin, and metronidazole therapy. Intravenous antibiotics are only indicated if there is evidence of pancreatic necrosis or if the patient develops a fever after the diagnosis of pancreatitis is made. There is a substantial amount of clinical literature validating this approach to treating pancreatitis. The appropriate sequence of events is to draw blood cultures prior to initiating therapy in order to maximize chances of detecting an organism.




A 24 hour old male infant is noted to have some peculiar jerking movements of the right foot and arm. His axillary temperature an hour before was 36.2"C. The physical examination reveals no unusual findings except that he appears small and premature. His birth weight was 2,550 g. The mother's and infant' s history show that he was the second of twins born after 37 weeks' gestation, presented in transverse position and his heart rate had dropped to 80 per minute 10 minutes prior to birth, with documented fetal hypoxia. He had an Apgar score of 3 at 1 minute and 7 at 5 minutes. The mother had nausea and vomiting during pregnancy for which she was given vitamin B6. She had mild preeclampsia at delivery.
*** Which is the most likely diagnosis?
a) Brain tumor
b) Hypoxemia in utero and possibly during delivery
c) Cerebral trauma during delivery
d) Vitamin B6 dependency
e) None of the above
*** In the diagnostic work-up, you may obtain the following tests or procedures, EXCEPT:
a) CT scan of the head
b) Lumbar puncture for examination and culture of spinal fluid
c) Electroencephalography
d) Serology for toxoplasmosis
e) Blood levels of sugar and calcium
B & D
Hypoxemia is the leading cause of seizures considering the history of drop in fetal heart rate and low Apgar score at 1 minute. Hypoglycemia, hypocalcemia, vitamin B6 dependency, and meningitis are possible causes of seizures but unlikely and should be ruled out. Some cerebral anomaly is possible. Incidence of congenital malformations is higher in twins than , in singletons, and central nervous system malformations lead all others in frequency. Brain tumors at this age are rare and usually present with recurrent vomiting and not with seizures.



An infant who was healthy at birth is brought to your office for her first office visit at the age of 6 weeks. You notice that the infant is jaundiced and that there is bilirubin staining of the wet diaper. Which one of the following diagnoses is most consistent with these findings?
a. Physiologic jaundice of the newborn
b. Hemolysis secondary to Rh incompatibility
c. Crigler-Najjar syndrome
d. Gilbert's syndrome
e. Biliary atresia
Answer is e- biliary atresia. bilirubbin in urine a clue of direct hyperbilirubinemia



A 22 y/o man used illicit IV drugs briefly 1 year ago. He is asymptomatic and has no history of viral hepatitis. Serum ALT and bilirubin concentrations are normal. Serologic studies show:
Hepatitis B surface antigen (HBsAg) positive
Hepatitis B core antibody (HBcAb) positive
Hepatitis B e antigen (HBeAg) negative
Antibody to surface antigen (Anti-HBs) negative
Antibody to e antigen (Anti-HBe) positive

Which of the following statements best describes the current condition?
a-He is in the incubation period, is highly infectious, is likely to develop acute hepatitis B.
b-He is in the incubation period, is minimally infetious, and is likely to develop acute hepatitis B.
c-He is a chronic carrier of hepatitis B and is highly infectious.
d-He was infected previously with hepatitis B. has recovered and is immune to hepatitis B.

Answer is a.

A 35 y/o woman requests a routine health assessment. She has no specific medical complaints or past medical hstory. Her father developed colon cancer at age 54 but is still living. His brother died of colon cancer at aae 41. Their father (the patient's grandfather) died in his sixties of colon cancer. His sister died of gastric cancer. The patent's sister was recently diagnosed with endometrial cancer at age 43.
The patient has a good appetite and has no nausea, vomiting, or weight loss. She has had no change in her bowel habits or hematochezia, and her menstrual periods are normal. Physical examination is normal. Digital rectal examination is negative for fecal occult blood. A screening complete blood count and biochemical profile are normal.
Which of the following actions is most appropriate?
a. Perform annual occult blood tests of three stool specimens. Initiate screening with flexible sigmoidoscopy at age 50.
b. Perform colonoscopy at least every 2 years.
c. Barium enema now; if normal no further evaluation.
d. Perform colonoscopy now; if no polyps are seen, no further investigation is necessary.
e. Perform fecal occult blood testing and flexible sigmoidoscopy now. If no polyps are seen now, the patient should be followed routinely with fecal occult blood testing and flexible sigmoidoscopy beginning at age 50.
Answer is b.




A 20 y/o woman is evaluated for fever of nine weeks™ duration. During this time she has had daily temperature elevations to 40.0 C (104.0 F). The only other symptoms have been malaise and occasional aches in the hands and knees; on one occasion she noted a transient pink rash on the abdomen. A one-week course of ampicillin had little effect.
T=38.9 C (102.0 F); PR=108 per minute, and rhythm is regular. A grade 1/6 systolic ejection murmur is heard best at the LSB. The spleen is palpable 3 cm below the left costal margin on deep inspiration. Electrocardiogram and chest radiograph are normal. Laboratory studies:
Hct 35%
Hgb 11.5 g/dL
WBC 12,800/cu mm; 81%neutrophils,
4% monocytes, 14 lymphocytes,
1% eosinophils
ASO 250 Todd units {<200}
RF Negative
ANA Negative
Blood cultures Pending
Which of the following is the most likely diagnosis?
(A) Enteric fever
(B) Bacterial endocarditis
© Lyme disease
(D) Hodgkin's disease
(E) Adult-onset Still's disease
Answer is e.



A healthy 71 y/o man describes visual loss in his right eye. Flashes of light and a curtain-like loss of lateral vision began when he awoke eight hours ago. These symptoms have persisted. Which of the following is the most likely explanation?
(A) Retinal vein occlusion
(B) Retinal detachment
© Atheroembolic occlusion of a lateral branch of the right retinal artery
(D) Ocular migraine
(E) Occipital lobe seizure
Answer is B.



What is the marker for CREST syn. ?

A. anti-scl-70
B. anti centromere
C. anti ds-DNA
D. SS-A (Ro)

The marker for CREST is anti centromere. They both have C in it.

anti-scl-70 is for scleroderma diffuse type. They both have scl in it.

D. anti ds-DNA is for SLE.

E. SS-A is for Sjogren's disease


What™s the characteristics of rapidly progressive glomerulonephritis (RPGN)?

A. crescent formation
B. "lumpy-bumpy" subepithelial deposits
C. IgM mesangial deposition
D. Associated with Hep C

the answer is A. Rapidly progressive glomerulonephritis is characteristic of rapidly losing at least 50% of its glomeruli in as short period of time--usually days to 3 month max. Extensive fibrinoid necrosis is found on biopsy.


Pt. In mVA was brought to ER, you suspected cardiac tamponade what is next step?
a. pericardiocentesis
b. echo first..
C. CXR
d. CT

Answer: depending on senario if pt is unstable proceed to Pericardiocentesis, If stable echo first.


A patient has end stage pancreatic cancer and as her primary care pysician, she asks you how long she could survive. What you should tell her.
a. 5 year rate is 30%
b. 5 year rate is 20%
c. " 15%
d. " < 5%

Answer: D- The overall 5-year survival rate for this disease is less than 5%.


A 24-year-old white primigravida has developed several 1- to 2-mm erythematous papules on her abdomen in the third trimester. They are pruritic and tend to appear in her striae. Liver function tests and a CBC are normal.

Which one of the following is the most likely diagnosis?

a. Pruritus gravidarum
b. Spangler's papular dermatitis
c. Impetigo herpetiformis
d. Herpes gestationis
e. Pruritic urticarial papules and plaques (PUPP)

The ans is e

The findings in this patient are most consistent with PUPP. This condition is usually benign, is not associated with increased fetal morbidity, and resolves after delivery, and there is usually no recurrence in subsequent pregnancies. Herpes gestationis, impetigo herpetiformis, and Spangler's papular dermatitis have different presentations and may be associated with increased fetal morbidity. Pruritus gravidarum is characterized by pruritus without skin lesions.


Ultrasonography reveals placenta previa in a 41-year-old asymptomatic G4P3 at 21 weeks gestation. Appropriate management would be

a. weekly speculum examinations under aseptic conditions beginning in her third trimester to assess the risk of bleeding
b. an MRI scan, with a repeat scan later in the pregnancy if indicated
c. repeat ultrasonography in her third trimester
d. cesarean delivery at 28 weeks gestation if her L/S ratio is favorable
e. reassurance that ultrasound diagnosis of placenta previa without evidence of bleeding is no cause for concern and can be disregarded

The ans is c

The incidence of placenta previa ranges from 6% to 45% in the second trimester, but more than 95% of these resolve by the third trimester. However, it remains a cause for concern and should be watched, not ignored, even if there is no bleeding. This patient should have repeat ultrasonography in her third trimester. An MRI is very helpful but need not be used except in a difficult diagnostic situation. Weekly speculum examinations would create a risk of hemorrhage. Delivery at 28 weeks would not be appropriate in a patient with no symptoms and without confirmation of the persistence of placenta previa.




Which one of the following is an absolute contraindication to tocolytic treatment for preterm labor?
a. Urinary tract infection
b. Documented gestation less than 28 weeks
c. Chorioamnionitis
d. Uncontrolled diabetes mellitus
e. Any vaginal bleeding due to mild abruptio placentae

The ans is c
Before tocolytic treatment is instituted, absolute contraindications to tocolysis must be ruled out. These include chorioamnionitis, severe abruptio placentae, severe bleeding from any cause, severe pregnancy-induced hypertension, fetal death, fetal anomaly incompatible with life, and severe fetal growth retardation. Chorioamnionitis may precipitate preterm labor and is an absolute contraindication to tocolysis. It may be present in a febrile pregnant patient even with intact membranes. In this case amniocentesis may be required to rule out infection. There are also a number of relative contraindications. These include uncontrolled diabetes, hyperthyroidism, maternal cardiac disease, mild chronic hypertension, mild abruptio placentae, stable placenta previa, fetal distress, fetal anomaly, mild fetal growth retardation, and cervical dilatation greater than 5 cm. In patients with relative contraindications to tocolysis the risk of complications from prematurity must be weighed against the risk of tocolysis. Not all vaginal bleeding is due to a serious obstetric condition. Cervical effacement or dilatation may be the cause. Even if the source of bleeding is determined to be a placental abruption, if the bleeding is minor, the abruption is mild, and the fetus is not in distress, tocolysis is not absolutely contraindicated.
While diabetes mellitus may be adversely affected by beta-adrenergic tocolytic agents, it is not an absolute contraindication to tocolysis. Close glycemic monitoring is, of course, mandatory. Even a few weeks of effective tocolysis may significantly alter the perinatal outcome of gestations between 25 and 27 weeks. Gestational age less than 28 weeks is therefore not a contraindication to tocolysis. While a urinary tract infection may precipitate preterm labor, tocolysis is not contraindicated. The infection, of course, should be treated.



Which one of the following statements is most accurate concerning juvenile rheumatoid arthritis?
a. Fever is a rare systemic manifestation
b. Ten years after the onset of disease, most patients have excellent functional status
c. Most patients have a permanent deformity of at least 1 extremity
d. The disease is characterized by lifelong recurrences
e. Most patients require corticosteroid treatment

answer is b.
At least 50% of patients followed for up to 15 years have complete remission of juvenile rheumatoid arthritis, and 70% regain normal function. A few patients are left with crippling joint deformities, but 75% have no significant residual deformity. Systemic-onset disease is accompanied by high fever, rheumatoid rash, polyarthritis, and other systemic manifestations




A 5-year-old African-American male fell off his bicycle and hit the back of his head on a hard surface. There was no loss of consciousness. No other injury was noted. He was obviously agitated and restless, and his only complaint was a loss of vision. When you see him, his examination is unremarkable except for moderate swelling over the occipitoparietal area of the scalp. His skin is intact, and no gross neurologic deficit is noted except for the visual loss. A CT scan is negative. An EEG shows only slight slowing of activity.
Which one of the following statements is true regarding this patient?

a. The child's vision will probably return within 24 hours
b. The child should be hospitalized for 72 hours for further observation
c. It will be months before the child's vision returns
d. The loss of vision is probably caused by damage to the optic nerve

Ans is a
Transient cortical blindness following mild head trauma is usually associated with a benign outcome. The special features are mild head trauma, no loss of consciousness, onset of blindness occurring within hours of the trauma, duration of blindness less than 24 hours, absence of skull fracture or visible injury on CT scan, and no other neurologic deficits. The EEG shows initial slowing with normalization on follow-up.



which of the following medication is known to exacerbate psoriaisis?
a)prednisone
B NSAID
c hydroxychloroquine
d methotrexate

answer is c.



1- A 20 y/o man who moved to the US from Cambodia 2 years ago presents to the ER with the sudden onset of left hemiparesis. His wife, who emigrated with him, reports that he has not felt well for at least a month and has had a weight loss of about 9 kg (20 lb). On P/E, T=38 °C (100.4 °F) and BP=116/52 mm Hg. He is somewhat cachectic and hemiparetic. The cardiac examination shows a murmur. The physical examination is otherwise normal.Which of the following tests is most likely to give the diagnosis?
(A) Blood cultures
(B) Lumbar puncture
© Radiography of the chest
(D) Complete blood count, differential, platelet count, and erythrocyte sedimentation rate
(E) Biopsy of inguinal lymph node
(F) head CT scan
Answer is A.The appearance of focal neurologic signs in a young person from Cambodia raises a wide differential diagnosis. Atherosclerotic cerebrovascular disease is uncommon in the 20-year-old age group. Among other diagnostic considerations are an embolic event from a cardiac source, such as a valvular vegetation associated with endocarditis or an atrial myxoma; vasculitis; tuberculomatous or bacterial brain abscess; brain tumor; aneurysm or arteriovenous malformation; and coagulation disorder such as thrombotic thrombocytopenic purpura or hyperviscosity syndrome caused by multiple myeloma.
Neurologic complications occur in 25% to 40% of patients with infective endocarditis. About 15% develop cerebral emboli with associated neurologic symptoms. The easily associated triad of new focal neurologic deficits, fever, and changing heart murmur occurs in only 33% of patients, so infective endocarditis must be considered in any patient with sudden focal neurologic deficits who has no conventional risk factors, such as atherosclerosis.


2- A 35 y/o woman presents with a 1.5-cm Lt breast mass which is nodular with indistinct borders. Her mother and maternal aunt both had breast cancer in their forties.
How would you evaluate this woman's condition?
(A) Observe through a menstrual cycle
(B) Diagnostic mammogram
© Breast ultrasound
(D) Office needle aspiration
This woman has a significant risk for breast cancer. The suspicion of cancer is high because of the characteristics of the mass and the family history of early-age breast cancer. The first approach would be a diagnostic mammogram to further define the lesion and, equally important, to look for suspicious lesions in the opposite breast. Even if the mammogram is negative, referral to a surgeon for biopsy would be the next step.


3- A 22-year-old female primagravida is seen in prenatal clinic 5 months after her last menstrual period. Her pregnancy has been uneventful, and she has gained weight progressively over the last 2 months. Three months ago, her blood pressure was 120/80 mm Hg, and there was 2+ protein on dipstick urinalysis. Today she has a blood pressure of 150/95 mm Hg and marked bilateral lower extremity edema.
Laboratory studies:
Hematocrit 29.7%
Leukocyte count 4200/L
Platelet count 209,000/L
Blood urea nitrogen 15 mg/dL
Serum creatinine 1.1 mg/dL
Serum uric acid 6.0 mg/dL
Urinalysis:
Protein 4+
Microscopic Hematuria, rare erythrocyte casts, rare broad casts, few leukocytes
24-hour urine protein excretion 12.5 g
Creatinine clearance 80 mL/min
This clinical presentation is most consistent with:
(A) Preeclampsia
(B) Underlying renal disease present before conception
© Hypertensive nephrosclerosis
(D) Pyelonephritis
Answer is B. In this 22-year-old pregnant woman, it is critical to determine whether previously existing underlying renal disease is present because of the overlap with signs of preeclampsia and the differing prognoses of the two conditions. Preeclampsia is a complication that presents after 20 weeks of gestation, which suggests the abnormal proteinuria in this patient is associated with another disease. The finding of 2+ protein on dipstick urinalysis is not specific, but the hematuria and erythrocyte casts suggest underlying glomerulonephritis rather than preeclampsia.


4- A 50-year-old man who has had diabetes mellitus for 12 years is concerned about becoming dependent on dialysis. His disease has been inadequately controlled (hemoglobin A1C range 8.7% to 11.8%). His urinalysis is strongly positive for protein, and his serum creatinine concentration has increased from 1.2 to 1.9 mg/dL in the past 8 months. His blood pressure has been 150/90 mm Hg and on occasion as high as 210/120 mm Hg. Which treatment is most important to reduce the process of this patient's kidney disease?
A. Insulin
B. Antihypertensive agent
C. HMG-CoA reductase inhibitor ("statin")
D. Protein-restricted diet
E. Aspirin
Answer is B. This patient has already started on the path to progressive diabetic nephropathy. Although all of the preventive measures still warrant attention, blood pressure control at this stage is most important. From the onset, good glycemic control (hemoglobin A1C 7% to 7.5%), a protein-regulated diet to reduce intraglomerular pressure as well as antihypertensive therapy with an ACE-inhibitor should be considered.Antihypertensive therapy attenuates the decline in renal function in patients with all forms of diabetes.

5- A 26-year-old patient is presented with amenorrhea of 2.5-month duration. Lab analysis indicates increased levels of thyroid and cortisol binding proteins. There is an elevated total cortisol and ACTH. Free cortisol level is within normal range. These findings are most likely suggestive of which of the following conditions?

A. Hypothyroidism
B. Addison's disease
C. Conn's disease
D. Cushing's disease
E. Pregnancy
answer is E.

Two weeks following a viral illness, a teenage boy breaks out in an evolving rash that is remarkable for target lesions. What is the primary treatment?
a. Epinephrine
b. Glucagon
c. Corticosteroids
d. Antihistamines
e. Symptomatic or supportive therapy depending on severity.
Answer is e. it™s erythema multiform

The most prevalent of allergic disease in school-age children is:
a. Atopic dermatitis
b. Food allergy
c. Asthma
d. Allergic rhinitis
e. Drug allergy
Answer is d.

Cheilosis and glossitis are features of ( more than one answer ):
a. vit A def.
b. riboflavin def.
c. vit. C def.
d. pyridoxine def.
e. vit. E def.
Answers are b&d.
Which one is the most common adverse effect of intranasal steroids?
a. Nasal irritation
b. Septal perforation
c. Nasal bleeding
d. Short stature
e. Adrenal suppression
Answer is a.

Increased risk for intussusception was observed as a rare complication following immunization with which vaccine?
a. IPV
b. OPV
c. Rotavirus vaccine
d. HAV
e. HBV
Answer is c.


A 60 y/o Japanese man visiting US with excellent health until 6 months ago, when he first noted mild upper abdominal fullness after meals. On P/E hyperpigmented, heaped-up velvety lesions in the neck, axillae, and groin is noted. Which of the following conditions is associated with the skin findings?

a- Non-Hodgkin's lymphoma

b- Anorexia nervosa

c- Acute leukemia

d- Adenocarcinoma of the stomach

e- Addison's disease

Answer is d, skin lesion is acanthosis nigricans.

Which of the following should be done annually after age 40 in the asymptomatic, average- risk man in order to promote the early detection of cancer?

a- Colonoscopy

b- Sigmoidoscopy

c- Digital rectal examination with palpation of the prostate

d- Digital rectal examination with palpation of the prostate and stool guaiac

e- Digital rectal examination with palpation of the prostate, stool blood test, and chest x-ray

Answer is c.

During a routine checkup, a 65 y/o man is found to have a level of serum Alk Ph. three times the upper limit of normal. Serum Ca and ph. concentrations and LFT results are normal. He is asymptomatic. The most likely diagnosis is

a- metastatic bone disease

b- primary hyperparathyroidism

c- occult plasmacytoma

d- Paget's disease of bone

e- osteomalacia

answer is d.


Which of the following is NOT a predisposing factor for the development of a hernia?
a- Ascites
b- Obesity
c- Cystic fibrosis
d- Chronic obstructive pulmonary disease
e- Peritoneal dialysis

answer is b. Ascites, peritoneal dialysis, ventriculoperitoneal
shunt, cystic fibrosis, and chronic obstructive pulmonary disease all predispose patients
to hernia formation because they increase intraabdominal pressure. Other risk factors
include a positive family history, undescended testis, and genitourinary abnormalities.



A 5-month-old infant has had several episodes of wheezing, not clearly related to colds. The pregnancy and delivery were normal; the infant received phototherapy for 1 day for
hyperbilirubinemia. He had an episode of otitis media 1 month ago. There is no chronic runny nose or strong family history of asthma. He spits up small amounts of formula several times a day, but otherwise appears well. His growth curve is normal. An examination is unremarkable except for mild wheezing. Which one of the following is the most likely diagnosis?
A) Benign reactive airway disease of infancy
B) Cystic fibrosis
C) Unresolved respiratory syncytial virus infection
D) Early asthma
E) Gastroesophageal reflux

Answer is E, gastroesophageal reflux is a common cause of wheezing in infants. At 5 months of age, most infants no longer spit up several times a day, and this is a major clue that the wheezing may be from the reflux. Also, there is no family history of asthma and the wheezing is not related to infections. Cystic fibrosis is more likely to present with recurrent infections and failure to thrive than with intermittent wheezing.




*** ECG findings of pulmonary edema include all of the following except:
a- deep S1
b- depressed ST in lead I & II
c- prominent Q1 & inversion of T3
d- left axis deviation
e- clockwise rotation in the precordial leads

answer is d. it cause Rt. axis deviation.



*** chest pain & friction rub 3 days after admission to ICU, indicate which of the ffg?
a- misdiagnosis of infarction
b- chest trauma
c- viral infection
d- transmural infarction
e- dissecting aneurysm

answer is d.


*** Neuropathy secondary to gout manifests as:
a- NS
b- ARF
c- ATN
d- Isosthenuria & moderate albuminuria
e- Malignant HTN

Answer is d.


*** Tx of choice for cutaneous manifestation of protoporphyria is:
a- phenobarb.
b- Corticosteroid
c- High carb. Diet
d- Beta caroten
e- Chlorpromazine

Answer is d.


*** 25 y/o female with intermittent double vision, on CXR an Ant. Mediastinal mass is noted. What™s the next step of Mx?
a- serum Ca messurement
b- brain MRI
c- evaluation of T cell function
d- serum gamma globulins messurement
e- order GTT

answer is d. association of thymoma and myasthenia gravis. 5-10% of Px with thymoma have low serum gamma globulins.


*** 76 y/o male with 80 pack/y smoking Hx, was diagnosed with lung cancer 4 m ago, he™s brought to hospital in state of coma, his serum ca= 16 mg/dl, which of the ffg, would be the most useful to reduce ca rapidly:
a- acetazolamide
b- furesmide
c- hydrochlorothiazide
d- manitol
e- spironolactone

answer is b- furesmide which increase urinary excretion of ca.



*** 50 y/o man with Hx of smoking & cough for the past 2 months, is found to have a solitary pulmonary nodule with no mediastinal dis on CXR. He™s expected to have best prognosis if he has which type of ffg tumors?
a- small cell carcinoma
b- poorly diff. Adenocarcinoma
c- metastatic carcinoma
d- well diff. SCC
e- well diff. Adenocarcinoma

answer is e- well diff adenocarcinoma



*** 27 y/o woman presents with sudden pain & decrease in hearing in Rt. ear while picking it with matchstick. On P/E, a traumatic rypture of Rt. tympanic membrane & some blood clot is seen. What™s the best advice or procedure?
a- advice her to use earpl
Reply
#2
U r doing a gr8 jpob dude
Reply
#3
thanks man.
Reply
#4
1. propranolol=drug of choice for HTN+beningn essential tremor


2. )Reccurent chalazion=> do histopathologic examination because of risk of Squamous cellular carcinoma (SCC)


3. )PH=7.23, HCO3=16, PCO2=40-what is it?
=mixed metabolic+respiratory acidosis, because if it were simple metabolic acidosis, then PCO2 would have been=1.5*HCO3+8=1.5*16+8=32mmHg and in this case it is 40mmHg, so there is also a respiratory acidosis


4. Know the formula for calculating the compensation in acidosis: PC02=1.5*HCO3+8
Ex.: a diabetic suffering from COPD treated with metformin may have mixed acidosis, metabolic from metformin, respiratory from COPD.


5. )NSAID can give SIADH (innapropriate secreation of ADH)


6. )Quinsy=peritonssilar abcess


7. )Tamsulosin=blocks alpha1-receptors only in prostate and bladder=>no side effects like hypoTN, headache, rhinitis as other alpha 1 blockers


8. )Sideroblastic anemia-high Fe, high TIBS, may be hypocromic and normocromic anemia in the same time=HIGH YIELD


9. )Streptoccocus bovis endocarditis-asoc. with colorectal cancer=>do colonoscopy


10. )Treatment of MG crisis=intubation+withdrawl of anticholinesterasic drugs for many days, then do plasmapheresis and iv IG


11. )Foreign body in the eye=>first pen light exam=>fluoresceine exam, if still (-)=>CT or US, never MRI (it can dislocate the foreign body)


12. )patient with XYY karyotype=severe acne, but not precocious puberty
21-hydroxylase deficiency=precocious pseudo-puberty, independent of the hypotalamic-hypophyseal-gonadal axe


13. )Black widow spider bite=>treat with narcotics, muscle relaxants and Ca gluconate


14. )Women with CIN (cervical intraepithelial neoplasia):
a) imunocpmpromized
b) in utero exposure to DES
c) hystory of CIN II/III
will have annual PAP smear done, regardless the normal previous ones.


15. )In severly depressed patient, even suffering from terminal illness with suicidal thoughts=>start antidepressant therapy


16. )Systemic steroids=drug of choice in sarcoidosis with disabling symptoms, if not responding=>cyclosporine
if asymptomatic sarcoidosis=>no treatment


17. )Lachman test is more sensitive than anterior drawer test in ACL rupture (anterior cruciate ligament)


18. )Primary polydipsia=increased thrist first, can be given by antipsyhotic medication


19. (phenotiazines) because they give a dry mouth. Look for a patient in the psychiatric ward on antipsychotics who gets diarhhea, polyuria and thrist
Insipid diabetes=increased polyuria first


20. )HTN+peripheral vascular disease=>give Ca channel blocker


21. )BCC (basal cell carcinoma)=most common tumor of the eyelid
=pearly, indurated
=>treat by chemosurgery of frozen section control excision


22. )Respiratory alkalosis-in liver insufficiency because of increased levels of progesterone (not metabolized anymore)


23. )Hashimoto thyroiditis-can be eu/hypo/hyper thyroid
-anti preoxidase Antibodies are diagnostic
-risk (6 times) of thyroid lymphoma

24. )Nasopharingeal carcinoma-appears in all age groups


25. )Colonic villous adenoma, sessile adenoma or size>2.5 cm=> increased risk of malignancy=HIGH YIELD
No further work-up for hyperplastic polyps


26. )US of KUB (kidney, ureter, bladder)=first step in evaluating BPH (benign proastate hypertrophy) with elevated Creatinine and normal urinalysis


27. )MALT gastric lymphoma and no metastasis=>eradicate H.pylori infection, if this fails, then give chemotherapy (CHOP)


28. )Athlete's foot (in swimmers too)=fungal infection, treat with tolnaftate


29. )Excessive bleeding (like abruptio placenta) in pregnant woman Rh (-)=>give higher dose of anti-D globulin


30. )If HDL>60mg%=>it removes one risk factor for CV disease


31. )Alcohol withdrawl=>first-line give chlordiazepoxide (librium), then lorazepam=second-line


32. )Sjogen syndrome=>diagnosis confirmed by lip biopsy (lymphoid foci in accessory salivary glands)=most specific


33. )Allergic contact dermatitis=erythema, edema, pruritus, tiny vesicles, weepy&crusted lesions 24-48h after contact with the allergen-cell mediated hypersensitivity


34. )Psammoma bodies=thyroid papillary cancer
Invasion of tumor capsule and blood vessels=folicular cancer
Hurtle cells-appears in both the above types (rarely)

35.
)Carcinoid syndrome=>develop deficiency of niacine (dermatitis, diarhea dementia) because Trp is mainly converted to serotonine and 5HIAA
Hyper Ca crisis=vomiting, oliguria/anuria, dizziness, coma


36. )Invasive aspergillosis="halo sign" on CT scan of lung
=in imunocompromized people
=on CXR-cavitary nodule


37. )Miastenia gravis=>thymectomy if:
-between age of puberty and 60
-if <12 years of age=>frecquent spontaneous remissions
-if>60 years of age=>give corticosteroids
-if only ocular disease=>no thymectomy


38. )Defect in Le adhesion=reccurent bacterial infections
-characteristic are: delayed separation of umbilical stump
necrotic periodontal infections


39. )Defect in opsonisation=asplenia (infections with S. Pneumoniae are frecq)


40. )Acute ingestion of masssive alcohol=> reduces the toxic effect of acetaminophen by inhibiting CYP2E1 like in suicide attemps with alcohol and acetaminophen together, but
Chronic ingestion of alcohol reduces glutathion=>increases the toxicity of acetaminophen like in an alcoholic patient with some kind of fever due to common cold


41. )Egg allergy=>contraindication to influenza vaccine and Q fever vaccine, and relative c.i. to MMR (which is however still recommended)-HIGH YIELD


42. )ACE inhibitors=>increase renin and kinin derivates=>cough


43. )Dermatomiositis=Gottron's sign=scaly patches on MCP and IP joints
=heliotrop rash (periorbital edema+purplish suffusion)
=proximal muscle weakness

44. )Vertebral osteomyelitis-lumber area frecq.=>back pain
- low fever, high ESR, local tenderness, spasm
Do MRI=of choice for diagnosis
-Complications=epidural abcess, spinal cord compression

45. )Pale lesions, velvet-pink or whitish that do not tan and are not scaly=tinea versicolor (Malassezia furfur)
-thick budding spores, large blunt hyphae="spaghetti and meatballs"
-treat=>Se sulfid shampoo and ketoconazole


46. )Loss of gastric fluid by NG suction in intestinal obstruction=>metabolic alkalosis (contraction alkalosis) even if initially it was a metabolic acidosis from the obstruction

47. )Hypophosphatemic rickets-only P is low, the rest are normal (PTH, Ca)

48. )Migratory trombophlebitis-due to cancer=> do CT of chest abdomen and pelvis; can be pancreatic, lung, prostate, stomach cancer


49. )Acyclovir=nephrotoxic=>renal tubular obstruction, gives crystalluria
)Deficiency of Iron=most common anemia in the elderly=HIGH YIELD
)Anemia of chronic disease is given by inflamatory joint disease not degenerative joint disease

)Pseudomonas osteomyelitis (punctures through snikers in the foot)=>give quinolones+surgical debridement
) Anemia of prematurity-normocitic, normochromic, low reticulocytes, few precursors of red line, normal WBC, T, normal Bilirubin, , no other abnormalities

)Isoniazide=>lowers the levels of GABA in the brain=>seizures in 1h after administration, treat by Vit. B6
)For screening use=total cholesterole+HDL
For treament use=LDL+Risk assessment

)Undiagnosed pleural effusion=best evaluated by thoracocentesis, except in CHF (here give diretics and see what happens-it will disappear)-HIGH YIELD
Varicose veins with incompetent perforating veins- can give:
-non-pitting edema
-medial leg ulcer
-fatigue
-brown discoloration of the ankles

)Warfarin induces skin necrosis=>mostly in patients with deficiencies of prot C or S or when it is started in high doses without prior heparin coverage (like in Atrial fibrillation)

)Larger Confidence interval (CI)=wider range of possible effects for a tested drug (efficient drug in some patients, non efficient in others)
If CI of 2 groups overlap=>no statistical significance
If they don't overlap=> statistical significant differences

)Niacine treatment=> raises HDL, but gives pruritus and flushing (release of Prostaglandines)=>prevent this by giving aspirin 30 min. prior to taking niacine

)Pulmonary embolism (PE)-post surgery in pacients with JVD and new RBBB; also dilated pupils=ominous sign

)Post-ictal (after seizures) lactic acidosis is transient=> no treatment, it resolves by itself in 60-90 min.
HCO3 is given in acidosis only if PH<7.2

)In primary hyper aldosteronism, there's no edema, but hyper Na

)Retinal vein occlusion=disk swelling, venous dilation and tortuosity, retinal hemorrhages, cottol-wool spots
)Retinal artery occlusion=pale disk, cherry-red fovea, boxcar segmentation of blood in the retinal veins

)Celiac disease-diarrhea, pallor (anemia), bone pain (osteomalacia), easy buising (low vit.K), hyperkeratosis (low vit. A), Ig A Antib. to gliadine, to endomissium and to tissue transglutaminase
=malabsortion+Iron def. anemia

)Overflow incontinence-can be given by: TCA, antichol., antipsychotics, sedative-hypnotics

)Spinal cord compression=>first give STEROIDS, then do MRI of the spine, if not available do=>CT myelogram=> give radiotherapy for malignancies=HIGH YIELD

) Diagnosis of Bartonella Henselae is clinical, confirm by Warthin-Starry stain of tissue specimen or Antib.
treat by azytromycin for 5 days and only in regional lymphadenopathy or systemic symptoms (also can give claritromycin, rifampin, TMP/SMX, cipro or Doxi)

)Shy-Dragger syndrome=Parkinson +autonomic instability (HypoTN...)
-can have bulbar dysfunction or laringeal stridor
-treat by iv expansion (fludrocortisone), salt, alpha agonists
Riley-Day Syndrome-AR disease, in Askenazi Jews
=autonomic dysfunction +severe hypoTN

)Prepubertal vulvo-vaginitis=>pruritus mainly in the night=>do scotch tape test to detect pinworm (do not answer stool examination)
)Treatment for low HDL=>Choice=fibrates, the niacine
)Treatment of choice for anorexia=hospitalisation

)Subcutaneous emphysema in patients on 100% O2 on mask=> do first CXR to rule out pneumotorax; if (-) this is a benign condition=>no furthet treatment

)Thoracic outlet syndrome-after MVA, playing with musicalinstruments, chronic illnesses, cervical ribs, congenital muscle band...=has signs of neuro-muscular bundle compression
-do EAST test for screening (elevated arm stress test), and CXR, MRI or angiography for confirming the diagnosis

)Medial meniscus tear-C shaped meniscus (the lateral is O-shaped)
by twisting the leg, effusion forms in 24h and is not as bloody as in ACL tear, tenderness on the medial part on the knee
=characteristic=bucket handle tear and locking of the knee joint in extension
(+)McMurray sigh=snapping felt with tibial torsion, knee flexed at 90 degrees
-trea by immobilization+bracing

ACL tear=hystory of hyperextension
(+) Lachman test, ant. drawer test, pivot shift test

)CI=1.02-2.15
RR=1.6 What does it mean? Well, the CI does not include value of where RR=1, then it means that the result is statistical significant
RR=1=> no effect or association

)Low cardiac output + high PCWP+high SVR=left ventricular failure (cardiogenic shock)=HIGH YIELD, I got some similar questions on actual exam, know this type of Q for cardilogy very well!!!

)Eczema herpeticum=HSV infection associated with atopic dermatitis, there are vesicles over the areas of atopic dermatitis
treat by acyclovir in infants

)Causes of altered mental status in the elderly patients:
low+high Na, Ca, low Mg, low P, low glucose, stroke, cardaic events, infections
-risk of dehydration by:WATER ACCESS IS DENIED=main mechanism in:
a)nursing homes
b)post-surgery
c)intubation in ICU

)To slow down the progression of Diabetic nephropathy=>restrict proteins and give ACE inhibitors (even if normal TN, but not if Clearance of Creatinine>2)

) Treat of both acute and chronic anal fissures=> starts with dietary modifications+stool softner+local anesthetics; in refractory cases do lateral sphincterotomy or gradual dilatation=>risk of incontinence and disruption of sphincter

)Paroxismal nocturnal hemoglobinuria-may cause pancitopenia
-like hemolytic anemia (High reticulocytes)
-intravascular process (high LDH, total billirubin, low haptoglobin)
-splenomegaly, Cooms (-)
-loss of iron in the urine=>microcytes, hyprocromia
-bone marrow=hypocellular
-flow cytometry=absence of CD59 -used for confirmation of disease

)Screen for hepatitis C if blood transfusion was before 1992 and for hep. B is before 1986

)To supress lactation=>tight-feeting bra+ice packs, no longer bromcriptine

)First step in organo-phosphoric poisoning=remove clothes and wash patient, then atropine
)Use CA-125+vaginal US for screening the intermediate risk of ovarian cancer in women with family hystory of ovarian cancer (not other type of cancer); otherwise, there's no screening done for ovarian cancer

)Avascular hip necrosis-given by steroids, scikle cell disease, alcoholism, SLE, Gaucher's disease
=hip pain with normal range of motion and normal X-rays. Do MRI=GOLD STANDARD for diagnosis
)Intraperitoneal rupture of bladder in MVA=previous full baldder in a patient who drank water (2l) and didn't urinated for 8h
Extraperitoneal rupture=more common

)Treat Ig A deficeincy:-prophilactic TMP/SMX, donor erytrocytes washed five times or bloodfrom other IgA def.patients

)HTA+osteoporosis=first choice are THIAZIDIC DIURETICS

)Vitiligo=pale macules with pigmented borders-acral or peri-orificial areas, autoimmune distrcution of melanocytes
Piebaldism=absence pf melanocyes, obvious from birth
Infection with M. Leprae=areas of hypopigmentation+anesthesia

)Metabolic alkalosis:
a) Cl sensitive (Urinary Cl<20mEq/l)-causes;vomiting,diurectics...=ECF contraction; treatable by NaCl infusion
b) Cl resistant (U Cl>20mEq/l)=ECF expansion, not correctede by Na Cl infusion; causes:persistent mineralocorticoid stimulation

)Nerve IX neuralgia-associated with Multiple Sclerosis (MS)

)Nephrotic syndrome-associated with arterial+venous thrombosis (freq. renal vein thrombosis and even PE
-anemia microcytes, hypocromia, resistant to iron therapy (loss of transferine)
-vit D deficiency, low Thyroxine levels

)Angiofibroma in the nose-can give bony erosions
Chondroma of nasal cartillage-is very rare in young patients

)Lyme disease in pregnancy and in children<9 years of age=>give AMOXI, not Doxi; also can give Azytromycin or cefuroxime

)Sympathetic ophtalmia-ant. uveitis, panuveitis, papillary edema, blindness
-by uncovering of "hidden antigens" (auto-Antib, cell-mediated reaction)

)Diphenilhydramine toxicity-seizures+cholinergic effects
Mercury overdose=vomitting, abdom. pains, bloddy diarrhea, renal insufficiency

)Best prevention of osteoporosis=HRT, c.i. if hystory of endometrial/breast cancer, but not if only family hystory

)Pulmonary nodule with cartillage in it=hamartoma=>observe
Radiotherapy-use it emergently in Superior Cava Vein syndrome

)Dupuytren contracture-associated with alcoholism, epilepsy, DM, TB, Peyronnie disease, Riedel thyroiditis; over 50 years of age

)Perforated retro-cecal appendix, edematous cecum, pus behind the ascending colon=>do right hemicolectomy+ileo-transverse anastomosis=best post-surgical results

)Excruciating pain from femur fracture=>give iv. Morphine (even in patients with hystory of drug abuse), then give PCA (patient controlled analgezia)
Give methadone in chronic severe pain syndromes

) If a test is (-)=> probability of the disease is =1-NPV

)HCO3 if given in lactic acidosis can depress the myocardium and increase the production of lactic acid by stimulating the phosphofructokinase enzyme

)Most common location of ischemic colitis is splenic flexure, than recto-sigmoid area

)MCC of bleeding in patients with renal failure is platelet dysfunction; treat by DDAVP

)Bacterial meningitis-treat empirically with ceftriaxone+vancomycine
Give iv dexamethasone in intracranial HTN and bacterial meningitis in infants

)The decision of using N-acetyl-cysteine for acetaminophen overdose is based on the drug levels taken at 4h post-ingestion. But if taken>7.5g acetaminophen or levels are not available at 8h of ingestion=>start treatment

)COPD exacerbation with: ph=7.32, pco2=52, po2=60=> give NIPPV (non-invasive positive pressure ventilation)-the indications are:
ph<7.35
pc02>45mmHg
Resp. rate>25/min.

)Adult Still's disease-variant of RA
-at 20-30 years of age
-high spiking fever
-salmon colored rash along with thye fever
-arthralgias, lymphadenopathy, high Le

)Any gunshot under the 4th i.c. space=>do laparotomy of the abdomen

)Latex allergy-associated with spina bifida
-gives anaphilactic reaction to "sex and surgery" (because of condoms and surgical gloves)

)Selection bias=loss of follow-up in a prospective study

)Reccurent myocardial infarction-detect by serial CK-MB levels

)RTA 1-assoc. with Sjogren syndrome
-urine ph>5.5
low HCO3
low K+

RTA 2-osteomalacia
-urinary ph<5.5
low HCO3 levels
-low K+

)Corrected Ca=measured Ca+0.8 (4.5-measured albumin)
For each 1g of lost albumin, Ca goes down by 0.8-1mg%

)Cavernous sinus thrombosis-like orbital cellulitis but with cranial nerves involvement (III,IV,V, VI) and bilateral
-treat by Antibiotics, then anticoagulation and corticosteroids

)Acute pancreatitis and non-alcoholic=>suspect gall-stones=>do US=first step; do CT only if Le>20000/ml and suspect necrotising pancreatitis

)DM foot ulcer-treat by cephalosporine, ampi/sulbactam, clidamycine+fluoroquinolone

)Syringomyelia=areflexic weakness + dissociated anesthesia in a "cape" distribution in the upper extremities

)Emergent contraception=estradiol+norgestrol, 2 tb taken in no later than 72h, 12h apart

)HIV individuals-give Td vaccine

)Aspirin sensitivity syndrome=pseudo-allergic reaction; treat with leukotriene inhibitors=drug of choice

)To confirm Ankilosing spondilitis=>do X-ray of the sacro-iliac joint, if not conclusive=>do CT
)Splenic rupture- the need of surgery determined by:
a)vital sighns+hemodynamic stability
b)change in hematocrit over time
c)need for blood transfusion

)NTG-dilates the capacity vessels not the resistance vessels=>reduces preload


)Molluscum contagiosum-poxvirus
-diseminated in HIV patients
-central umbilicated, dome shaped
resolves spontaneous in 1 year

)Bullous myringitis-painful vesicles on the timpanic membrane
-mycoplasma or viral infection

)Drug-induced pancreatitis: valproic acid, diuretics (loop and thiazidic), 5-ASA, sulfasalazine, imunosuppressive (L-asparaginase, azathioprine), AIDS-patient (didanosine, pentamidine), metronidazole, tetracycline

)Cystinuria-reccurent stones since childhood
-family hystory
-hexagonal crystals, hard stones which are radio-opaque
-screening test=urinary cyanide nitroprusside test

)Antiphospholipid syndrome-reccurent fetal losses, reccurent artery+vein thrombosis
-types-I has false (+) VDRL
-II has lupic anticoagulant=>false + APTT
-III-has anticardiolipin antibody
treat in pregnancy by: heparin+aspirin and measure factor X activity and not APTT for treatment

)Gilbert syndrome-associated with fasting, alcohol, stress, complete reversal with phenobarbital
Crygler Naijar 1-billirubin=8-30mg%, kernicterus, no response to phenobarbital
Crygler Naijar 2-billirubin<20mg%, no kernicterus, 25% response to phenobarbital

)In gonochoccal arthritis=>do urethral culture to identify the gonococcus

)Common variable imunodeficiency=15-35 years of age
-normal B and T cells
-frecq. sino-pulmonary infections
-low IgG, Ig M, Ig A
Wiskot-Aldrigh=has low IgM, but high Ig A, IgE

)Acute appendicitis-can perforate and give pelvic abcess with diarrhea, fever, tender mass on rectal exam, low abdom pain and not the usual sighns of appendicitis

)PTCA=has better outocme than thrombolysis

)NPV is high and PPV is low if the pre-test probability of the disease is low

)Microalbuminuria in DM=>start ACE inihibitors even if no HTN

)Motility disorder of the oesophagus=>do contrast study=>then oesophagoscopy to exclude mecanic causes (strictures,cancer)=>only then manometry

)Chronic headaches+painless hematuria=>think analgesic nephropathy due to papillary necrosis

)Terminal patients with severe pain=>give short-acting morphine, then long-acting narcotics

)Campylobacter jejuni-MCC of diarrhea
-in undercooked poultry
-diarrhea is watery or hemorrhagic

)Craniopharingiomas-bimodal distribution:
-children (gives growth problems)
-55-65 age group (gives sexual dysfunction, bitemporal hemianopsia-think of a truck driver who has to turn his head all over when looking sideways

)Chlamydia screening-do it in all women under 25 years of age if sexually active

)Nonseminomatous tumor-may give ant.mediastinal mass, high AFP, HCG; treat by cysplatinum for 6 weeks; use the above two markers to monitor the therapy

)Signs of high ICP: dilated pupils, anisocoria
flaccidity, decerebrate or decorticate posturing
papilledema
NOT Glasgow coma scale=it assesses the severity of head injury only

)Matching=tool that makes cases and controls have similar distribution of some important confounding variables; it's an efficient mean to control confounding

)Warfarin induced skin necrosis-pain, bullae, skin necrosis; use heparin until they heal

)Furosemid-is ototoxic

)SLE-has-low T supressor cells, high T helper cells>B-cell hyperactivity>high serum Antib and Ig G auto-anitbodies which form the immune complexes

)Hyperlipidemia 1 and 5=associated with pancreatitis

)In PCP in HIV patients give steroids, besides TMP/SMX if:
-Pa O2<70mmHg
-A-a gradient>35=150-(1.25*PCO2)-PO2
sat O2<75%

)Amantadine-dual drug=anti-viral+anti-Parkinson

)Granulosa cell tumors-precocious puberty
-postmenopausal bleeding

)Trachoma-given by chlamydia trachomatis (A-C)
-cause of blindness by neovascularisation=pannus
-follicular conjunctivitis
-treat by oral erytro or tetracycline

)Anserine bursitis-medial knww pain below the joint line, hystory of trauma
-valgus stess test is (-), it's (+) in medial collateral ligament strain
-X-ray of tibia is normal

Patellofemural syndrome-in females <45 years
-ant. knee pain aggravated by flexion
-retropatellar tenderness and crepitation

)A narrower CI=>the study is more precise

)If DM type II is not controlled by one hypoglicemic agent, add another one from another class; give insulin only if BUN and Cr are abnormal

)In acute bacterial prostatitis=>get urnie sample for culture before starting empirical antibiotics

)Acute variceal bleeding-give octreotid; beta-blockers are for prophylaxis

)Mucormicosis-treat by surgical debridement and amphotericin B

)Labetalol=drug of choice in pregnancy if HTN+DM nephropathy

)Open-angle gluacoma-cupping of optic disk
-loss of peripheral vision=tunnel vision
-more frecq. in African-Americans
Macular degeneration=central vision loss

)Give MMR to all HIV patients (except those severly compromized)

)Patients with impaired consciousness, advanced dementia=>predisposed to aspiration pnemonia due to impaired epiglotic function


)In patients with frecq. attacks of gout=>first step is to measure 24h urinary uric acid level
-<800mg/day=>under-secretion (probenecid)
->800mg/day=>over-production (allopurinol)

)Ulnar nerve syndrome=MC site of entrapment is: medial epicondilar groove; think of counter clerks who sit with their elbows on the table all day

)In CHF-improved survival by: aspirin, beta-blockers, ACE inhib., spironolactone
-don't improve survibal: digoxin and loop diuretics

)What acid-base disorder is this?
ph=7.53
pco2=30mmHg
HCO3=24
Cl=85
Na=138
Well, tough one: we have alkalosis and is respiratory because Pco2 is low;
AG=Na-(Cl+HCO3)=29=>metabolic acidosis;
But a change in the AG (increase) is accompanied by a similar change in the levels of HCO3 (decrease). Here HCO3 is normal=> metabolic alkalosis.
Scenario (all three together): pneumonia=>respiratory alkalosis
vomiting=>metabolic alkalosis
DKA (from ketones)=>metabolic acidosis

)Any patient with bone pains, renal failure and hypercalcemia has Multiple Mieloma until proven otherwise

)Sudden RUQ pain+rise in hematocrit levels with hepatomegaly, splenomegaly and ascites=>think Budd-Chiari syndrome (from polycitemia vera)=>first step=hepatic venogram or liver biopsy

)Carotid stenosis of >60%-99%=> do CEA even if asymptomatic; complete occlusion=c.i. to surgery

)Lichenus sclerosis-dryness, severe itch, vaginal soreness
-may give vaginal cancer
-do biopsy to rule out cancer
-treat by topical steroids

)Retinal artery occlusion-treat with occular massage+high flow O2
-give thrombolytics within 4-6 hours of visual loss

)TCA- cardiotoxic potential=>because they inhibit fast Na channels
-to asses the severity of the toxicity=>get QRS duration

)Pneumovax-has capsular polysacharrides and gives T-cell independent B cell response; live vaccines give T-cell dependent......

)All uncouncious patients, even those breathing need airway established by:
-intubation in the ICU
-cricothyroidectomy in the field

)In unstable angina=> no thrombolytics; give aspirin, heparin, NTG, beta-blockers

)In MI, give thrombolytics after sublingual NTG to rule out vasospasm.

)VF=is a reentrant ventricular arrythmia

)Superficial spreading melanoma=MC
-increased intraepithelial atypical melanocytes
Acral lentiginous melanoma-on palms, soles, beneath nail plate
Lentigo melanoma-head, neck, arms of fair skinned older people

)Diuretic use (also thiazidic)=>gives metabolic alkalosis

)Euthyroid sick syndrome: low T3, Normal T4, normal TSH; in severe disease: low T4+T3, normal TSH


)Cholesterol embolism-follows surgical or intervention on arterial tree
-livedo reticularis, gangrene, ulcer or mottling of toes
-systemic eosinophilia, low complement levels
-renal failure (eosinophiluria)

)If an ulcer is seen on colonoscopy=>do biopsy to rule out cancer

)Subconjunctival hemorrhage-benign condition, observe only; think of a person with a red eye on awakening in the morning

)Parvovirus infection=>arthralgias of small joints with 5-10 min morning stiffness, normal ESR, no signs of inflamation locally
-joint involvement is symmetrical+-rash

)Intermitent claudication-give aspirin and exercise program
-do angiogram only pre-op,otherwise do duplex arterial study if you want imaging

)Acantosis nigricans-insulin resistance (DM)-in young people
-gastro-intestinal malignancy in the older
-symmetrical, hyperpigmented, velvety palques in axilla, groin, neck

)Drug-induced acute allergic interstitial nephropathy:
a)antibiotics (meticilline, cephalosporine, sulfonamides...)
b)NSAID
c)thiazides
d)phenytoin
e)allopurinol

)Epiglotitis-given MC by H.influenzae type B and strep. group A

)Food droolong out of the mouth and nose during meals=Zenker diverticulum=> can give aspiration pneumonia
-diagnose by oesopphagography

)Chronic diarrhea=>oxalate stones (due to malabs. of fatty acids, they bind Ca and oxalate is free for absortion)

)Gardner syndrome:colonic polyps+lipoma, nasal angiofibroma, gastric polyps, osteomas, epidermoid cysts, more teeth

)Amebic hepatic abcess-"anchovy paste" in the liver
-treat by oral metronidazole (not percutaneous drainage)
-in the tropics acquired

)Cluster headache=>treat by 100%O2 and s.c. sumatriptan; prophilaxis: verapamil, Li, ergotamine

)Vaginal delivery in breech:
-frank or complete breech
->36 weeks of gestation
-weight:2500-3800g
-adequate maternal pelvis
External cephalic version-converts breech into cephalic presentation=>use over 37 weeks until the onset of labor
Internal podalic version-in twin delivery (from transverse/oblique to breech)

)Friedrich ataxia-ataxia, dysarthria, skeletal deformities (scoliosis, hammer toes, pes cavus), cardiomyopathy; three words: neurologic, cardiac and skeletal problems

)Pseudogout-can be triggered by surgery or trauma
-may have 100000 Le/ml in joint fluid

)Nosocomial infections-UTI
-surgical wound infection
-pneumonia

)Know the X-ray appearance of descented aorta aneurysm-well circumscribed lesion=>due to aterosclerosis

)For acute aortic dissection=>use first TEE or CT(only if hemodynamically stable)

)Herpetic withlow-HSV 1 and 2
-in health care workers
-pain in the finger pulp with vesicles and systemic symptoms
-treat by acyclovir+topical bacitracin (to prevent secondary infection)
Felon-appears in tailors
-from needle injuries
-it's a bacterial abcess (tense abcess)=>do drainage+cephalosporine

)Urinary diversion procedure (ureter implanted in the ileum for example)=>gives Hyper Cl metabolic acidosis because the colon absorbs NH4+ derived from ureea under the action of intestinal bacteria
-also the pum Cl/HCO3 functions like this: absorbs Cl, lets HCO3 go!

)Tinitus-by aspirin, quinine
-also in Meniere disease, acoustic neuroma and ...depressed patients=>give TCA

)Chronic myelogenous leukemia=>give IMATINIB (Gleevec)=Tyrosine kinase inhibitor; side effects are: nausea, diarrhea, cramps, rash, face swelling, temporary reduction in blood cell production

)Post-herpetic neuralgia=>give acyclovir
-follows acute herpetic-zoster infection

)Intra-uterine fetal demise (IUFD)-death in utero>20weeks
-Beta-HCG remaind eleveted
-confirm with US
-first do a coagulation profile to assess the risk of DIC
<=>between 13-28 weeks, no DIC=>watchful expectancy
<=>between 13-28 weeks, with DIC=>induction of labor with PG suppositories
<=>after 28weeks, with/out DIC=>induction of labor with oxytocin and laminaria tents
-can use vaginal delivery
So, in case of DIC=>deliver immediately by induction of labor

)Acute retinal necrosis in HIV patients
-pain, keratitis, uveitis
-peripheral pale lesions+central retinal necrosis
-given by HSV or VZV
CMV retinitis=hemorrhages+granular lesions around the retinal vessels

)Pneumonia after upper GI endoscopy=>suspect anaerobic bacteria=> give clindamycine or ampi+metronidazole
Ampi+genta=used almost NEVER for pneumonia, they are used for abdominal infections+-metronidazole

)Cyclophosphamide=> can give bladder cancer
-prevent by lots of fluids and mesna

)Stress fracture (March fracture) or insufficiency fracture:
-young adults who exercise a lot
-X-ray=normal, dignose it with CT or MRI
-dull pain increased by exercise
-point tenderness
-at metatarsal, navicular, neck of femur/tibia

)Hypothyroidism-consider in patients with unexplained high CK levels

)Bicuspid aortic valves=>can give aortic stenosis later in life (4-5th decade)=MCC of aortic stenosis in middle-aged adults

)Seborrheic keratosis-in the elderly
-0.3-2cm large
-slow enlargement
-greasy surface, stuck on appearance-HIGH YIELD these words
-varies in color
-anywhere on the body, except palms and soles
-can itch or rub
-do shave biopsy (DON'T DO IT FOR MELANOMA)
)Aspirin toxicity-gives metabolic acidosis and respiratory alkalosis (not normal compensation, but two distinct processes)

)Gastroparesis-treat in order by: metoclopramide, bethanecol, erythromycine; also cysapride-but it gives cardiac arrythmias
-confirm study=Nuclear Medicine Scintigraphy

)To determine the type of jaundice (conjugated or unconjugated) do:
-check urnary excretion of billirubin
-Van der Bergh test

)Lyme disease prophylaxis-do it only in pregnant women with hystory of tick bite in order to RELIEVE ANXIETY by Amoxi
-give vaccine to people living in moderate/high risk areas
-to the rest of the population=NO PROPHYLAXIS

)MCC of SAH (subarahnoid hemorrhage) syndrome in children is AVM rupture
-may have a case with hystory of seizures and migraine-like headaches

)Screen patients for lipid abnormalities:
-men>35 years
-women>45 years, if in good health or
between 20-35 (M) and 20-45 (W) if they suffer from:
a)DM
b)family hystory of hyperlipidemia
c)personal risks of CAD
d)family hystory of cardiac disease<50 years (M) or <60 years (W)

)Gout attacks-give:
-low-purine diet
-no alcohol=>metabolises to lactic acid=>impairs renal excretion of uric acid by acidifying the ph
-avoid diuretics, pyrazinamide=>they compete with uric acid for renal excretion

)Penile fracture=emergency
-penis deviated to one side due to the rupture of albugineea of a cavernous body
-first do retrograde urethrogram, then surgical exploration of the penis

)Hypercarotinemia-in anorexia, DM, hypothyroidism

)A confounder (ex.smoking), to be considered so, needs to be linked to the:
-exposure (ex.people who drink alcohol are more likely to smoke)
-outcome (ex. smoking is associated with oral cavity cancer)
So smoking can be a confounder if a study concludes that alcohol alone is responsable for oral or oesophageal cancer!!!

)Effect modification=the effect of exposure (ex.drugs) on outcome (a disease) is modified by another variable (ex.family hystory). This effect is not BIAS!!!

)Amiodarone=> don't give it if low BP
)Lesion suspicious of melanoma=>do excisional biopsy with narrow margins

)Radioactive iodine=treat of choice for Grave's disease
Antithyroid drugs-give in pregnancy+Grave's
-pre-op for surgery on Grave's
Iodinated contrast agents=>treat thyrotoxicosis
-give them if intolerant to anthytiroid drugs
Surgery-if:
-very large goiters
-antithyroid drugs do not control thyrotoxicosis in pregnancy
-increased risk of malignancy
-if next year scheduled pregnancy

)Petrous apicitis-triad=retro-orbital pain, paralysia of lateral rectus, otorrhea

)Anoscopy/proctoscopy=first step for blood per rectum in patients <50 years of age, without risk factors for colon cancer (here's not included blood mixed with stool=this is a risk factor)

)Sickle cell disease-aplastic crisis-prevent by folic acid administartion and NOT by vaccination against parvovirus (which is a nother cause)

)Exercise level in pregnancy=keep it at the same level as before preganancy

)Phenothiazine-gives hypothermia, inhibit shivering-think of a schizo wandering in the streets in winter and is found lying in a park

)If someone exposed to HBV is vaccinated already and HAS a documented response to HBV (by prior adequate titer of antibodies)=>do nothingm, just reassure!!!

)If in 2 weeks from the beginning of a pneumonia, the CXR is still pathologic=> do bronchoscopy and CT scan to rule out abcess or tumor and to take cultures, to drain...

)A reliable test gives similar results on repeat measurements
A valid (accurate) test gives results that can be compared with a gold standard test!!!

)Graft versus host disease=> affects skin (rash), intestin (blood+diarrhea), liver (abnormal LFT)
-by activation of donor T-cells, so it's cell-mediated immune response
)In DKA there's a paradoxycal hyper K+ from acisosis, because the reserves of K+ are actually depleted=> so give K+ in your treatment plan

)FIRST sign of colo-rectal cancer under 40 years of age=Rectal Bleeding

)Removal of K+ from the body=cation-exchange resin (kayexalate), diuretics, dialysis; NOT Ca gluconate, NOT insulin (these create only a shift intracellularly)

)Roth spots and Osler nodulesm in IE are from immune complex deposition (immune vasculitis)
Janeway lesions=septic embolism

)Optic neuritis=rapid vision decrease
-marked changes in color perception
-pain on eye movements
-central scotoma
-afferent pupillary defect
-swollen disk

)Myotonic muscular dystrophy
-begins in childhood (~13 years)
-AD disease
-muscle weakness, wasting in distal hands, post. forearms, ant. compartment of legs
-myotonia=delayed relaxation
-associated with testicular atrophy, DM, frontal balldness, hypothyroidism

)Matitis-in nursing
-treat by anti-Staph penicillin
-continue nursing to decrease breast engorgement and observe
-drainage only if there'sa fluctuating mass (=abcess)

)Atracurium-metabolized in plasma, by serum esterases
-use it safe in renal and liver dysfunctions
Pancuronium+mivacuronium=>not good in renal disease
Rocuronium=>not good in liver disease
Succinylcholine-in renal disease gives Hyper K+ and apneea

)Randomization=similarities in the baseline characteristics of patients in both placebo and treated groups. It controls known/unknown confounders!!!

)Mild acne=>topical retinoids; if reactivation occurs, give topical antibiotics
Moderate to severe acne (papular or inflamatory acne)=>oral antibiotics
Nodulocystic, scarring acne=>oral isotretinoin

)IMPORTANT!!!
In any metabolic acidosis=>first step=get AG (Na-Cl-HCO3); normal is 6-12 (or 8-14 depends on the author)
a)Normal AG metabolic acidosis-usually hyper Cl-2 causes:
-renal loss of HCO3 by RTA, moderate renal failure (GFR>20ml/min), carbonic anhidrase inhibitors
-GI loss of HCO3 by diarrhea, pancreatic fistula, ureterosigmoidostomy
To differentiate between them, get next step=urine AG(Na+K-Cl); normal is from -50 to 0.
If (+)=> defective urine acidification, lower urinary Cl, like RTA 1,2 and 4
If (-)=> higher urinary Cl=>adecquate NH4+ production=> GI causes
Then, urine PH differentiates different types of RENAL causes of acidosis!!!
b)High AG metabolic acidosis:
-lactic acidosis
-ketoacids (DM, starvation, alcohol ingestion)
-methanol ingestion
-ethjylen glycol ingestion
salycilate poisoning
uremia (GFR<20ml/min)
Here, next step is to calculate the osmolar gap (especially if toxicity is suspected)

)Paget disease=disordered skeletal remodelling
Osteoporosis=low bone mass with normal mineralization
Rickets=defective mineralization of bone and cartillage of the growth plate

)Malignant otitis externa-by pseudomonas (in DM, imunocompromized)
-granulations of the ear canal
-involvement of CN VII, IX to XII
-erosion of bone
-foul smelling discharge
-deep otalgia
Rhizopus+Aspergillus are fungal infections that give external otitis, (not malignant), but don't make granulations

)Suspect Zollinger-Elisson syndrome if refractory ulcers are associated with renal stones and there's a + family hystory of PUD

)HIGH-YIELD:
Erythopoietin in dialysed patients-side effects:
-raises BP, even HTN encephalopathy
-headaches
-flu-like syndromes
-red-cell aplasia (rare)
Start treatment at Hematocrit<30% or Hb<10g%

)Gall-bladder pathology-First use US
HIDA scan-for diagnosis of acalculous cholecystitis or suspected on US

)Trichinosis-2 phases:
a)GI upsets:pain, nausea, vomit, diarrhea
b)splinter hemorrhages, retinal&conjunctival hemorrhages, periorbital edema, chemosis, muscle pain, tenderness, swelling, weakness

)Eato-Lamber syndrome-earliest manifestation is hip-girdle weakness, later shoulder girdle involvement

)Thioridazine toxicity-cardiac arrythmias, prolonged QRS, low BP, tahicardia; treat by NaHCO3

)Sputum gram stain of pneumonia-purulent specimen if >25 neutrophils and <10 epithelial cells/field

)Respondent bias=when outcome is obtained by patient's response (ex.migraine) and not by objective means of diagnosis (ex.biopsy)

)Tinea corporis infection-itching
-ring-shaped scaly patches with centralclearing and distinct borders
-caused most frecq. by Trichophyton rubrum
-treat by topical terbinafine
-use griseofulvine if only extensive disease (this drug is usually not used today)


)Salivary glands inflamation-from drugs such as: tioureea, iodine, cholinergic drugs

)Hepatitis B-associated with membranous GN
Carrierrs of Hep. B virus-associated with membrano-proliferative GN

)Vit.K deficiency:
-NPO patient
-receiving antibiotics
-high PT, then high PTT (prolongation of PT>> prolongation of PTT)

)Primary sclerosing cholangitis-in Ulcerative Colitis
-complication of it is cholangiocarcinoma=contraindication to transplant
So any severe stricture of the biliary tree MUST BE BIOPSIED!!!

)Mitral regurgitation=MC valvular abnormality in patients with IE

)Hollenhorst bodies=cholesterol particles, signs of impending stroke

)Benzodiazepines-can cause paradoxycal agitation in the elderly

)Congenital cataract=MCC of white reflex in pediatric population

)Any nocturnal, newly diagnosed asthma in middle-aged patients=>suspicion of GERD if associated laryngitis; to differentiate between asthma and GERD give a trial of proton pump inhibitors which can be both dignostic anf therapeutic

)CREST syndrome-may have pulmonary HTN (loud P2 sound)
)Mechanical symptoms in patients with meniscal problems=>evaluate by arthroscopy or MRI, then correct by surgery (arthroscopic or open)

)Latent period-in chronic disease epidemiology; is NOT BIAS
=extended time of continuous exposure is needed to affect the outcome (ex. 2 years of continuous multivitamins administration is needed to give a protective effect against cancer)

)Nitrates are c.i. for at least 24h after taking sildenafil (viagra)

)SCC-scaly plaque, central ulceration, 1.5cm, on the forearm
-polygonal cells, atypical nuclei at all levels of epidermis, zones of keratinization; sometimes is difficult to tell it apart from BCC

)Hypo K+ metabolic alkalosis:
-vomiting=>low urine Cl
-S. Bartter
-S. Gitelmann
-diuretic abuse; all three have high urine Cl

)Otosclerosis-AD disease
-Women>>men
-more in Caucasian race
-treat by NaF

)Lactose intolerance-diagnose with Lactose Breath H+ Test or (+) Clinitest for stool reducing substances
-high osmotic stool gap=290-2(stool Na+stool K)>50mosm/kg

)Goodpasture's syndrome-give emergent plamapheresis!!!

)Metastatic bleeding brain mass=> FIRST think MELANOMA
Tumors that don't metastisize to the brain are:
a) non-melanomatous skin cancer
b) oropharingeal cancer
c) oesophageal cancer
d) prostate cancer


)Parkinson's tremor=>gibe benztropine
Choreea Huntington=>give haloperidol

)Vitreous hemorrhage-sudden, acute loss of vision
-sudden onset of floaters
-fundus is hard to visualize, floating debris, dark red glow
-treat conservatively-sleep in upright position

)Prematurity-cause of intraventricular hemorrhage

)Mamography-do it annually between 50-75 years
FOBT-anually; 50-80 years
sigmoidoscopy-every 5 years between 50-80 years
Pap smear-until the age of 65 years, not after
Lipid screening-men>35y
-women>45y
-not >75y
-??unknown recommendations between 65-75y

)Alpha-1 antitrypsine deficiency in non-smoker, 3rd decade of life, lower lobe emphysema, neonatal jaundice in hystory

)Relapsing polychondritis
-recurrent inflamation of cartillaginous structures: ear=MC,spares the lobules; eye (conjunctivitis, episcleritis); joints (diffuse joint pains); skin; CNS.

)Fracture of calcaneum due to fall=> evaluate for other potetial fractures by X-ray of head, chest, abdomen, lumber area and pelvis
It's a very painful fracture and prior taken narcotics may alter pain perception
-so give morphine in severe pain, but after investigating for potential injuries (head and neck first)!!!

)Li-can cause seizures, opisthotonus, hyperreflexia, coma

)Attributable risk percent (ARP)=risk in exposed-risk in unexposed/risk in exposed=1-risk in unexposed/risk in exposed=1-1/RR=RR-1/RR
-it's the excess risk in the exposed population attributable to the risk factor!!!

)Impaired NH4+ excretion=main mechanism of metabolic acidosis in renal failure

)Hypo K+ periodic paralysis-by stress or medication
-sudden drop in K+ levels
-renin=normal, BP=normal
-familial condition or thyrotoxicosis

)Bartter syndrome=hypoK+ metabolic alkalosis
-impaired Na absortion in Henle ascending loop=hypovolemia=>activate RAA syste
-polyuria, polydipsia, growth abnormalities
-high urine Cl

)Serous otitis media=>air bubbles seen in the middle ear
Cholesteatoma-marginal tympanic membrane perforation, ear canal filled with mucus, pus, granulation tissue, destroys bone; remove surgically

)All patients suspencted of Zollinger-Elisson syndrome should have checked:
-first: serum gastrin levels (if>1000pg/ml are diagnostic); if not diagnostic,
-thenConfusedecretin stimulation test, if (-),
-then: Ca infusion study
Also measure gastric PH once to exclude hypergastrinemia from achlorhydria

)Chlamydial urethritis-dysuria, urinary frecquency
-mucopurulent urethral discharge
-sexual hystory of multiple partners
-pyuria+absence of bacteria on urialysis (colonies<100/ml)
In gonochoccal, the discharge is purulent!!!

)Differentiate CML from leukemoid reaction by measuring Leukocyte Alkaline Phosphatase (LAP); low in CML, high in leukemoid reaction!!!

)Lewy body dementia-fluctuating cognitive impairement
-reccurent visual halucinations
-motor features of Parkinson

Pick's disease-fronto-temporal
-personality changes (euphoria, dezinhibition,apathy)
compulsive behaviour
-peculiar eating habits, hyperorality
-impaired memory
-visuo-spatial function=intact

)Transient synovitis of the hip:
-X-ray=normal
-2 weeks after URI
-high ESR
-pain of joint movements
-treat by bed rest and hip joint in the position of comfort

)Beta-blockers intoxication-treat by:
-atropine, if fails
-isoproterenol, if fails
-glucagon

)Gold standard for osteoporosis is DEXA scan

)Increased ventilatory rates onmechanical ventilation=>can lead to auto-PEEP=>lowers BP
-treat by decreasing the ventilator rate

)Erythema nodosum-may be the first sign of sarcoidosis
-goes with flare-ups of IBD
-appears in TB too

)Osgood-Schaltter disease-treat with rest, NSAID, brief casting

)Beta-blockers-selective beta 1 are agents of choice for perioperative MI risk decrease

)Cocaine abuse-young, venous trace marks, EKC with ischemia (ST depression), tahicardia, HTN
-treat-first line: Benzodiazepine, then nitrates, aspirin
-do catheterisation only if MI is obvious!!!

)Use aspirin in children only in:
a)Kawasaki disease
b)Juvenile rheumatoid arthritis

)Patients on both Mg(OH)2 and Kayexalate can get metabolic alkalosis due to the fact that Kayexalate bind Mg, so HCO3 remains in access and can be reabsorbed from GI tract!!!

)Treatment of hypo Na (SIADH):
-mild, asymptomatic, Na=120-130: fluid restriction
-moderate, asymptomatic, Na=110-120: normal saline+loop diuretic
-severe, symptomatic: hypertonic saline (3%)+loop diuretic

)Laxative abuse-10-20 evacuations/day and night,cramps=cause of factitious diarrhea
-on colonoscopy you see melanosis coli, because of the use of anthraquinone-laxatives (bisacodyl) and is dark-brown discoloration of the colon with shining limph follicles as pale patches

)Cyclosporine side effects=HIGH YIELD:
a)nephrotoxicity, high K+
b)HTN-tret with Ca channel blocker
c)neurotoxicity-tremors
d)glucose intolerance
e)infections
f)malignancy
g)gingival hypertrophy, hirsutism
h)GI complaints
Tacrolimus-same side effects except no gingival hypertr. and no hirsut.
Mycophenolate=>bone marrow suppression
Azathioprine=>bone marrow suppression,hapatotoxicity, diarrhea, leukopenia

)Pernicious anemia=>has high levels of LDH

)Nosocomial pneumonia in intubated patients=>think Pseudomonas=>give cefepime or ceftazidime (4th generation)

)A definite diagnosis for Alzheimer's disease can be made only post-mortem by brain biopsy!!!

)Chronic fatigue syndrome-like fibromialgia,but no trigger-points+symptoms of at least 6 months duration

)Barton's fracture-intraarticular, carp+distal margi of radius
Chauffeur's fracture-of radial styloid
Galeazzi's fracture-anywhere on the radius+radio-ulnar junction
Smith's fracture-reverse Colles fracture

)Null-hypothesis for cross-sectional study:
There's no association between elevated ESR level and colon cancer!
Null-hypothesis for cohort study:
The risk of colon cancer is the same for the subjects with and without elevated ESR level

)Descending aortic aneurysm in a young male is frecq. due to blunt trauma to the chest!!!
HIGH-YIELD:

)Bullous penphigoid=IgG+C3 deposits at the dermal-epidermal junction
-no oral lesions
-tense blisters in flexural areas
->60 years of age
-pruritus precipitated by UV, NSAID, antibiotics

Pemphigus vulgaris-intraepidermal blistering disease with auto-antibodies to adhesion molecules
-Ig G deposits intracellularly in the epidermis

Cicatricial pemphigoid-affectsmucous membrames
-Ig G deposits in linear band at the deromo-epidermal junction

Herpes gestationis-2nd trimester
-sub-epidermal blisters
-deposits of C3 at the basement membrane zone

)Amphotericine=>can give hypoK+

)Hshimoto thyroiditis:
-low TSH, high T4, T3, high T3 resin uptake
-low radioactive iodine uptake
-high thyroglobilin level
-non-tender goiter
-dry mouth&eyes

)Leukoplakia:
-increased risk of SCC
-from tabacco, vit. A, B deficiencies, syphilis
-do incisional biopsy or exfoliative citology examination!!!

)Soemtimes UC can involve the terminal ileum=>backwash ileitis

)Adult PKD complications:
-hepatic cysts
-valvualr heart disease (MVP, AR)
-colonic diverticula
-abdominal wall and inguinal hernia

)Thrombastenia Glanzmann-RA disease
-defect in GP IIb-IIIa
-increased BT
-trombocytes=normal, vWF=normal
-clot retraction is decreased
-epinephrine, colagen, thrombin and ADP fails to induce aggrgation
-normal response to ristocetin test
Bernar-Soulier syndrome-giant trombocytes, bleeding tendencies
-lack of aggrgation to vWF and ristocetin
-normal responde to ADP
-abnormality in GP Ib

)Acalculous cholecytitis:
-extensive burns
-severe trauma
-prolonged TPN
-prolonged fasting
-mechanical ventilation

)HIV patients with esophagitis-give fluconazole against Candida; if it donesn't cure, then and only then do esophagoscopy, cytology, biopsy, culture!!!

)In alkali ingestion(ex. lye)=>do contrast study with gastrografin+endoscopy as easrly as possible
-don't give charcoal=ineffective!!!

)Bupropion-can be given with nicotin patches, monitor BP=>risk of HTN
-causes weight loss
-risk of seizures=>don't give to anorexic patients as antidepressant

)In tumor lysis syndrome=>give allopurinol+hydration!!! If on allopurinol, give only 25% of purine antagonists (mercaptopurine, azathioprine)!

)In heat stroke=>rapid cooling with evaporating cooling=choice,then gastrci lavage or imersion in cold water

)Dermatitis herpetiformis=>dapsone

)In metab acidosis=>look for compensation (PaCo2)=>look at AG=>look at variation of AG and HCO3; one goes up and the other should go down, if this is the only acid-base problem, so the divisation of their variations is equal to 1!!!

)Prevalence high=>high PPV, low NPV

)Nickel jewelry, poison ivy=allergic contact dermatitis (type IV reaction)

)Do thyroid function tests if:
-hyperlipidemia
-unexplained low Na
-high CK levels

)Treat strep.pharingitis with one shot of benzatin-penicilline G i.m.

)MC complication of PUD=hemorrhage

)MC drug causing priapism=prazosin

)Asymptomatic lymphocytosis in older=>suspect CLL=> lymphadenopathy=>hypogamaglobulinemia=infections!
-Smudge cells are characteristic
-lymph node biopsy is not required for diagnosis, but it confirms it!

)In primary billiary cirrhosis-give ursodeoxycolic acid=first line, second-line=cholestiramine; ultimate cure=liver transplant



)Actinomycosis-anaerobic gram (+) branching bacteria
-draining infection, indurated area
-sulfur granules
-treat by high dose penicilline or erythromycine 6-12 weeks!!!
Another cause of draining face/neck infection=scrofula!

)Scoliosis-Cobb's angle
-mild curve<20degrees=>careful follow-up to assess the rate of progression
-20-30 degrees=>choice between observation and bracing is made on the presence of rotational deformity and + family hystory
->30 degrees=>bracing
->40degrees=> surgery

)Unacceptability bias=participants' response with desirable answers which leads to underestimation of the risk factors-ex. medical students are asked to complete a survey about whether they smoke or not...some may say that they don't smoke just because they know that smoke is bad for health!!!

)Hystory of normal skin at birth+gradual progression to dry scaly skin=ichthyosis
-dry skin with horny plates over the extensor surfaces of the limbs
-worsens in winter due to increased dryness=lizard skin!

)Markers of adrenal tissue: DHEA, DHEA-Sulfate
Testosterone+androstendione coem equally from ovaries and adrenal glands

)Symple renal cyst on CT=>reassurance (know the CT appearance), but investigate if:
-multilocular mass
-thcikened irregular walls and septae within the mass
-contrast enhancement

)In mononucleosis, patients can play sports only when physical exam is normal (no splenomegaly)

)TIA-give anticoagulation if embolic
-in the rest, give aspirin; if fails, give clopidogrel, if fails, give ticlopidine
-aspirine+dipiridamole-if there's a hystory of TIA on aspirin alone

)Zellweger's syndrome-defect in peroxisomes
-neonatal seizures
-facial dysmorphism
-hypotonia, wide open sutures, cloudy cornea, glaucoma
-they live only a few months

Neonatal adrenoleukodystrophy-no dysmorphic features
-enlarged liver, abnormal LFT, pigmentary degeneration of retina, impaired hearing

X-linked adrenoleukodystrophy
-accumulation of fatty acids (log chain ones)=>pregressive adrenal cortex dysfunction and CNS white matter

Infantile Refsum disease
-gait problems, hearing loss, pigmentary degen. of retina, dysmorphic features

Classic Refsum disease-young adulthood
-visual problems (night blindness), ataxia, cardiac arrythmia, ichthyosis, peripheral neuropathy

)Osteoporosis-acute backpain with noobvious preceding trauma in the elderly women=compression fractures of the vertebrae

)More than 5 days of symptoms of appendicitis with RLQ findings=>give iv. hydration, antibiotics (cover gram (-)), bowel rest

)Chronic therapy with Vit. D=>hyper Ca
-stop vit.D, low Ca diet, acidify urine, give corticosteroids!



)To control confounding-means:
-matching
-randomization
-restriction
Selection bias=>controlled by selecting a representative sample of the population for the study+high rates of follow-up
Ascertainment bias=>avoided by a strict protocol of case ascertainment
Observer's bias=>controlloed by blinding

)Patients with dyspepsia and <45 years of age without alarm symptoms (bleeding, anemia, dysphagia, weight loss)=>first step=Non-invasive test for H.pylori (breath test or serologic test)
-if +=>eradication therapy of H.pylori
-if -=>empirical trial of H2 blockers or PPI or prokinetics
If >45 years or with alarming symptoms=>do endospcopy+H.pylori testing

)Hematuria+irritative or voiding symptoms=>suspect bladder cancer, even in a patient with large, firm prostate
-associated factors: cigarette smoking, suprapubic pain, long hystory of analgezic use

)Chronic ITP-can be a feature of SLE; that's why in chronic cases of ITP=> do BM biopsy

)In conjugated hyper bilirubinemia=> first do LFT, then US of CT scan

)Dubin-Jonhsons+Rotor syndromes=> have normal ALP

)Hawthorne effect=tendency of the studied population to affect the outcome due to the fact that they are being studied!!!
Sample distorsion bias=when the sample is not representative for the whole population in "exposure" and "outcome"
Information bias=imperfect assessment of association between exposure and outcome as a result of errors in the measurement of exposure and outcome-it's minimized by standard techniques for surveillance and measurement+trained observers

)HIGH-YIELD:
Left ventricle dysfunction: high RA and PCWP pressure
Septic shock: low PCWP and RA pressure, low SVR, high debit
In pericarditis, tamponade=>RA pressure=PCWP and both are high
RV infarct=> low PCWP, low BP

)Post-prandial worsening pain+avoidance of food+risk factors for atherosclerosis=abdominal angina

)Hydatid cysts-Echinococcus granulosus
-asymptomatic
-mostly in the liver, but also lung
-comes from sheep
=fluid-filled cyst with numerous secondary daughter cysts
Pig farmers=> get neurocysticercosis
Commercial sex-worker=>can get perihepatitis from STD (gonorrhea)

)Congenital adrenal hyperplasia-hirsutism, virilization+very high levels of 17-HO-progesteron, normal Testosteron and DHEA
Idiopatic hirsutism-from excessive peripheral conversion of testosteron=>dihidro-testosteron

)If bleeding >25-30% of blood volume (or 1500ml)=>give blood transfusion
In trauma, usually give 2l of crystalloid solution in 10 min=>if still hypovolemic=>give blood
Give blood also if:
-Ht<25%
-Ht<30%+ COPD, Ischemic Heart, chronic renal failure
-unexplained acidosis in anemic patients

)Epiglotitis=>first do fiberoptic laryngoscopy to establish a diagnosis, then do nasotraheal intubation (alternative=traheostomy)

)HIGH-YIEDL-Know this thouroughly!!!
low PCWP, normal MV02 (mixed venous O2 concentration)=>is septic shock; also: low RA pressure, high debit, low SVR
low PCWP, low MVO2=>is volume depletion or neurogenic shock
)Atopic dermatitis-infants<6 months
-pruritus
-symmetrical-scalp, cheeks, trunk, extensor areas
-unknowun etiology
-in acute attacks give low-moderate potency corticosteroids
-spares diaper area (contact dermatitis does not)

Exfoliative dermatitis-over 40 years of age
-prior dermatological condition, systemic illness, new medication

)Dehydration in diabetes insipidus=>give normal saline; once the volume deficit is replaced, can switch to 0.45% saline to restore water deficit

)HIGH-YIELD
CT scan=>use it to evaluate the extent of newly diagnosed gastric cancer; treat by surgery

)In dialysis-persist or worsen the followings: anemia, HTN, bone disease, autonomic neuropathy

)Intrahepatic cholestasis of pregnancy
-3rd trimester with jaundice
-marked pruritus, high AST, ALT(<200), very high bile acid levels, ALP<200, PT=normal

)If suspect IUFD=>GET US, then coagulation profile

)Spontaneous hemarthrosis=>think hemophilia

)Osteogenesis imperfecta=>get type 1 collagen assay

)If one parent gives consent for the treatment of a minor is sufficient 9even if the other one disagrees)=>go ahead with the treatment!!!
If both parents refuse treatment=>get a court order if the situation is not emergent!!! If it's emergent=>treat as you see fit; you are protected by the law that you act in the best interest of the child!!!

)Inhaled steroids=> can give dysphonia, thrush

)In a ureteral colic, one can have intestinal ileus=>do CT scan or IVP; treat by ureterolithiasis
Enteroclysis=used to diagnose small bowel tumors or other pathology which cause intestinal obstruction

)In case-control studies, if prevalence of the disease is low=>odd ratio=~RR (this is rare disease assumption). They love these questions, try to understand them, though they are difficult!!!

)Study of choice for abdominal aortic aneurysm=abdominal US

)Malignant melanoma-MC as a changing mole
-hystory of sun exposure
3 major crteria:
change in-color
-shape
-size
4 minor criteria:
inflamation
bleeding/crusting
>7mm
sensory changes

)In metabolic alkalosis from vomitting=> give K, not Ca which is normal in total, only ionized is decreased

)In acute respiratory alkalosis=>PH raisis the afinity of Ca for albumin=>decreased ionized Ca, increased albumin-bound Ca, normal total Ca and normal Ca bound to inorganic anions

)Chronic pancreatitits with central duct>1cm and severe pain=>do lateral pancreatico-jejunostomy

)Alcoholism=>can cause rhabdomyolysis (drunk man lying in a park)=> urine dipstick is + for blood but (-) for RBC=>from myoglobinuria

)IE of drug users: give vanco+genta because of high incidence of meticillin-resistant Staph
-if not drug abuser=>give naph+genta

)Acarbose (alpha glucosidase inhibitor)=> don't allow carbohydrate breakdown in the intestin=>high carbohydrates in the stool!
-indicated in late onset type II DM

)Dysphagia to both solids&liquids=>achalasia=>domanometry, but alsoendoscopy to rule out cancer

)Acute monicitic leukemia- M5A in young (~16 years)
- M5B-middle age(~49 years)
-(+) alpha-naftyl esterase
-numerous promonocytes and monocytes
M3 leukemia-assoc. with DIC

)Post-op benign intrahepatic cholestasis
-after major sugery
-low BP
-extensive blood loss
-or after massive blood replacement

Halothane toxicity-type 1-mild elevation of LFT, no jaundice
-type 2-acute liver failure

)Kaposi sarcoma-on the trunk, face, extremities
-papules>plaques/nodules
Bacillary angiomatosis-like Kaposi sarcoma, but with systemic signs (malaise, headaches, fever)

)Prolactin is (+) by TRH, serotonin and (-) by dopamine
Hyperprolactinemia=>(-)GnRH=>amenorrhea
=>galactorrhea
Both of them appear in hypothyroidism due to high TRH

)Idiopathic pulmonary fobrosis-treat by steroids; lung transplant is not an option since it must be bilateral and there's a shortage a donors!

)In fibromyalgia=>check thyroid tests and CK levels
Diffuse axonal injuries-from deceleration
-at the gray-white matter junction
-looses consciousness on the spot=>then persistent vegetative state
MRI is more sensitive than CT for detecting the axonal injuries

)Lumbosacral strain-pain after exertion
-absence of radiation
-(+) paravertebral tenderness
-(-) straight-leg raising test
-normal neurologic exam

)Situational syncope: middle-aged, old male, losses counsciousness immediately after urination or during coughing fits

)Hypocalcemia-after major surgery, extensive transfusions
-due to volume expansion, hypoalbuminemia
Mypo Mg-alcoholism
-diarrhea, diuretic abuse
-prolonged NG suction
)Management of DM foot ulcer from mild to severe forms:
-off-loading
-debridement
-wound dressing
-antibiotics
-revascularization
-amputation


)In refractory ascites=>do tapping of up to 2l/day+give 10g albumin/liter taped as a last resort before any surgical procedure you may think!!!
Spleno-renal shunt-will worsen ascites
Side to side porto-caval shunt-improves ascites, worsens encephalopathy
Peritoneo-jugular shunt-for the treatment of ascites only; risksBig GrinIC, sepsis, peritonitis
iv furosemide therapy=>worsens encephalopathy, precipitates hepato-renal syndrome

)Treat Ca-oxalate stone lithiasis by:
a)increased fluid intake>3l
b)normal or increased Ca intake-yes!!! you read fine!!! (1000mg)
c)low Na diet
d)low oxalate diet (no chocolate, vit C)
e) low dietary proteins (meat, eggs)

)Soap-bubble appearance in the distal femur on x-ray with knee pain, some mass, in 20-40 years old=>think giant-cell tumor of the bone=>refer to orthopedic surgeon; don't do bone scan=this is for solid tumor metastasis identification

)Choledocal cyst=congenital anomaly of the biliary tree
-dilatation of the intra/extra hepatic ducts
-jaundice, acholic stools
-reccurent attacks of pancreatitis
-can degenerate into cholangiocarcinoma
-First do US, then CT or MRI
Caroli'ssyndrome=congenital
-intrahepatic dilation of bile ducts

)In transplant patients=> give TMP/SMX to prevent PCP pneumonia; alternatives:dapsone or aerosolized pentamidine

)Lumbar pain in the 3rd trimester=> due to lumbar lordosis and relaxation of the ligaments of the sacro-iliac joints

)Dystonia-treat by benztropine or diphenilhydramine

)90% of PE come from emboli in the ileo-femoral veins

)Pseudogout under 50 years=>think any of:
hypo Mg, hypo P, hyper PTH, hypothyroidism, hemochromatosis

)Treat fractures of the humeral shaft by: closed reduction and immobilization of the arm in a hanging cast

)Cystic fibrosis-may develop Cl sensitive metabolic alkalosis
Barter'syndrome-Cl resistant metab. alkalosis

)In GERD=> give empirical treatm. with PPI or H2 blockers=>if it fails or there are features of complicated disease=>esophagoscopy=>if (-)=>PH monitoring

)HIGH-YIELD
Acute tubular necrosis-muddy-brown cast
GN-RBC cast
Pyelonephritis, interstitial nephritis-WBC cast
Nephrotic syndrome-fatty cast
Chronic renal failure-broad, waxy cast

)Febrile reaction to transfusion=>prevent it by Le depletion with cell washing

)Chronic Hep C with normal enzymes=>follow yearsly with LFT, not biopsy!!!

)Biophysical profile (BPP); N=8-10
-if AFI<5=>deliver because of possible cord compression
-if BPP=6 and no oligohydramnios=>do CST, if non-responsive=>deliver; if suspicious=>repeat CST in 24h
-if BPP=4, no oligohydramnios and fetal lungs are mature=>deliver; if not mature=>give steroids and do BPP in 24h
-if BPP=2=>deliver no matter what!!!

)Cyclical vomiting-in children with parents with headaches
-no obvious cause

)Contraindications to exercise stress test are:
LBBB
ST>1MM
WPW Syndrome
idioventricular rythm
**but NOT: RBBB, ST depression at rest<1mm

)Ehylene glycol poisoning-has Ca-oxalate rectangular, envelope shaped crystals
-high AG=>SO WHAT DO YOU DO???
-Osmolar Gap=observed osm-calculated osm
calculated osm=2Na+Glu/18+BUN/2.8

)Chronic painful DM neuropathy: give TCA, NSAID or GABAPENTIN=CHOICE

)Constipation with back pain,
Reply
« Next Oldest | Next Newest »


Forum Jump: